Med Surg Exam 4 Chapter 53-59

Réussis tes devoirs et examens dès maintenant avec Quizwiz!

Which of the following form a part of home care instructions for a client with epididymitis and orchitis?

All options are correct

A 51-year-old client is being seen by an urologist for perineal pain, low back pain, fever lasting 5 days, and painful urination. The physician confirms a diagnosis of prostatitis and orders treatment. During your client education session, which of the following recommendations do you make?

All options are correct.

The nurse is conducting a health history when the client states that her last menstrual period was 6 months ago. Upon further questioning, the client also states that symptoms of hot flashes and mood fluctuations. Which question should the nurse ask next?

"Are you taking any hormone replacement therapy?"

The client with polycystic kidney disease asks the nurse, "Will my kidneys ever function normally again?" The best response by the nurse is:

"As the disease progresses, you will most likely require renal replacement therapy."

The nurse weighs a patient daily and measures urinary output every hour. The nurse notices a weight gain of 1.5 kg in a 74-kg patient over 48 hours. The nurse is aware that this weight gain is equivalent to the retention of:

1,500 mL of fluid

A nurse is obtaining health history from a young adult woman. Which of the following would alert the nurse to a possible problem?

Reports of dyspareunia

The nurse has been asked to provide health information to a female patient diagnosed with cystitis. Select all the teaching points that apply.

-Cleanse around the perineum and urethral meatus after each bowel movement. -Drink liberal amounts of fluid. -Void no more frequently than every 6 hours to allow urine to dilute the bacteria in the bladder. With an infection, fluids should be increased up to 4 L/day, but caffeinated beverages should be avoided because they can irritate the urinary tract. Therefore, voiding more than seven times per day will help clear out bacteria from the bladder. See Box 28-3 in the text.

Which nursing diagnosis is appropriate for the client with a new ileal conduit? Select all that apply.

-Deficient knowledge: management of urinary diversion -Disturbed body image -Risk for impaired skin integrity Deficient knowledge, disturbed body image, and risk for impaired skin integrity are expected problems for the client with a new ileal conduit. Urinary retention and chronic pain are not expected client problems.

A client is diagnosed with polycystic ovarian syndrome. Which of the following findings would the nurse expect to assess? Select all that apply.

-Hirsutism -Impaired glucose tolerance -Sleep apnea Polycystic ovarian syndrome is associated with obesity, insulin resistance, impaired glucose tolerance, dyslipidemia, sleep apnea, and infertlity. In addition, menstrual periods may be irregular. Menorrhagia (excessive bleeding) is more commonly associated with fibroids.

A client is prescribed amitriptyline, an antidepressant for incontinence. The nurse understands that this drug is an effective treatment for which reason? Select all that apply.

-Increases bladder neck resistance -Decreases involuntary bladder contractions Some tricyclic antidepressant medications (amitriptyline, nortriptyline, and amoxapine) are useful in treating incontinence because they decrease bladder contractions and increase bladder neck resistance. Anticholinergic drugs such as oxybutynin chloride (Ditropan) reduce bladder spasticity and involuntary bladder contractions. Bethanechol (Urecholine) helps to increase contraction of the detrusor muscle, which assists with emptying of the bladder.

A client is being treated for a malignant bladder tumor. What would be included in treatment of a small tumor? Select all that apply.

-resection and fulguration -topical application of an antineoplastic drug Small, superficial tumors may be removed by cutting (resecting) or coagulation (fulguration) with a transurethral resectoscope. Topical application of an antineoplastic drug may be used after resection and fulguration of a tumor. Cystectomy is a surgical removal of the bladder and is performed for large tumors that have penetrated the muscle wall. Urinary diversion is performed after a cystectomy.

A client has just undergone a urinary diversion procedure. What management issues related specifically to urinary diversion would be included in this client's care plan? Select all that apply.

-Observe for leakage of urine or stool from the anastomosis. -Maintain renal function. -Assess for signs and symptoms of peritonitis. Management issues related specifically to urinary diversion procedures include observing for leakage of urine or stool from the anastomosis, maintaining renal function, assessing for signs and symptoms of peritonitis, maintaining integrity of the urinary diversion and urine collection devices, maintaining skin and stomal integrity, promoting a positive body image, and teaching the client how to manage the diversion. Oral intake is important for any postoperative patient after it is approved by the physician; however, this is not specific to the care of the urinary diversion client.

The nurse is providing an education program for the nursing assistants in a long-term care facility in order to decrease the number of UTIs in the female population. What interventions should the nurse introduce in the program? Select all that apply.

-Perform hand hygiene prior to patient care. -Assist the patients with frequent toileting. -Provide careful perineal care. In institutionalized older patients, such as those in long-term care facilities, infecting pathogens are often resistant to many antibiotics. Diligent hand hygiene, careful perineal care, and frequent toileting may decrease the incidence of UTIs.

A 27-year-old patient is a regular smoker and is diabetic. She is requesting contraceptive therapy. She has been diagnosed with hypertension. She says she finds it cumbersome and time consuming to visit the doctor regularly just to check her blood pressure. Her vital signs are temperature 98.8°F, pulse 72, respirations 12, BP 112/72 mm Hg. In addition, laboratory reports show a hemoglobin A1C level of 5%. Which of the following aspects of patient teaching would the nurse recommend? Select all that apply.

-Purchasing a self-monitoring cuff -Discussing methods for stress reduction -Advising a smoking cessation program -Prescribing a combination oral contraceptive pill

A nurse who will be working at a health fair is preparing a presentation on reproductive health and contraception. Which of the following would the nurse expect to include as advantages of combined hormonal contraception? Select all that apply.

-Reduced incidence of benign breast disease -Improved acne control -Reduced risk of ovarian cancer -Decreased risk of anemia

The nurse practitioner performing a physical assessment on an 80-year-old woman is looking for which of the following functional changes in the reproductive system? Select all that apply.

-Thickening and stiffness of the vaginal wall -Uterine prolapse -Vulvar dystrophy -Vaginal dryness

Which of the following is not a definition of erectile dysfunction?

The inability to achieve an erection after sexual activity

The nurse is providing supportive care to a client receiving hemodialysis in the management of acute renal failure. Which statement from the nurse best reflects the ability of the kidneys to recover from acute renal failure?

The kidneys can improve over a period of months

The nurse who teaches a client about preventing recurrent urinary tract infections would include which statement?

Void immediately after sexual intercourse. Voiding flushes the urethra, expelling contaminants. Showers are encouraged, rather than tub baths, because bacteria in the bath water may enter the urethra. Coffee, tea, colas, alcohol, and other fluids that are urinary tract irritants should be avoided. The client should be encouraged to void every 2 to 3 hours during the day and completely empty the bladder.

Which of the following is a strategy to promote urinary continence?

Void regularly, 5 to 8 times a day Strategies to promote urinary continence include increasing awareness of the amount and timing of all fluid intake; avoid taking diuretics after 4 PM; avoiding bladder irritants, such as caffeine, alcohol, and aspartame (NutraSweet); taking steps to avoid constipation by drinking adequate fluids, eating a well-balanced diet high in fiber, exercising regularly, and taking stool softeners if recommended; and voiding regularly, 5 to 8 times a day (about every 2 to 3 hours).

The nurse reviews a client's history and notes that the client has a history of hyperparathyroidism. The nurse would identify that this client most likely would be at risk for which of the following? Kidney stones Chronic renal failure Fistula Neurogenic bladder

A A client with hyperparathyroidism is at risk for kidney stones. The client with diabetes mellitus is a risk factor for developing chronic renal failure and neurogenic bladder. A client with radiation to the pelvis is at risk for urinary tract fistula.

The nurse is reviewing the results of renal function studies of a patient. The nurse understands that which of the following is a normal BUN-to-creatinine ratio? a) 10:1 b) 8:1 c) 4:1 d) 6:1

A A normal BUN-to-creatinine ratio is about 10:1. The other values are incorrect.

Which nursing assessment finding indicates the client with renal dysfunction has not met expected outcomes? a) Client reports increasing fatigue. b) Urine output is 100 ml/hr. c) Client rates pain at a 3 on a scale of 0 to 10. d) Client denies frequency and urgency.

A Fatigue, shortness of breath, and exercise intolerance are consistent with unexplained anemia, which can be secondary to gradual renal dysfunction.

A client is prescribed flavoxate (Urispas) following cystoscopy. Which of the following instructions would the nurse give the client? a) "This medication will relieve your pain." b) "This medication will treat the blood in your urine." c) "This medication prevents infection in your urinary tract" d) "This medication prevents urinary incontinence."

A Flavoxate (Urispas) is a antispasmodic agent used for the treatment of burning and pain of the urinary tract.

The nurse is caring for a client who reports orange urine. The nurse suspects which factor as the cause of the urine discoloration? phenazopyridine hydrochloride Infection Metronidazole Phenytoin

A Orange to amber-colored urine is caused by concentrated urine due to dehydration, fever, bile, excess bilirubin or carotene, and the medications phenazopyridium hydrochloride and nitrofurantoin. Infection would cause yellow to milky white urine. Phenytoin would cause the urine to become pink to red. Metronidazole would cause the urine to become brown to black.

A client is scheduled for a renal ultrasound. Which of the following would the nurse include when explaining this procedure to the client? a) "You don't need to do any fasting before this noninvasive test." b) "You'll have a pressure dressing on your groin after the test." c) "A contrast medium will be used to help see the structures better." d) "An x-ray will be done to view your kidneys, ureters, and bladder."

A Renal ultrasonography identifies the kidney's shape, size, location, collecting systems, and adjacent tissues. It is not invasive, does not require the injection of a radiopaque dye, and does not require fasting or bowel preparation. An x-ray of the abdomen to view the kidneys, ureters, and bladder is called a KUB. A contrast medium is used for computed tomography of the abdomen and pelvis. A pressure dressing is applied to the groin after a renal arteriogram.

A patient has stage 3 chronic kidney failure. What would the nurse expect the patient's glomerular filtration rate (GFR) to be?

A GFR of 30-59 mL/min/1.73 m^2

The nurse is caring for a patient with dementia in the long-term care facility when the patient has a change in cognitive function. What should the nurse suspect this patient may be experiencing?

A UTI The most common subjective presenting symptom of UTI in older adults is generalized fatigue. The most common objective finding is a change in cognitive functioning, especially in those with dementia, because these patients usually exhibit even more profound cognitive changes with the onset of a UTI.

What is true about extracorporeal shock wave lithotripsy (ESWL)? Select all that apply.

-Stones are shattered into smaller particles that are passed from the urinary tract. -ESWL is administered with the client in a water bath or surrounded by a soft cushion while under light anesthesia or sedation. Stones are shattered into smaller particles that are passed from the urinary tract. ESWL is administered with the client in a water bath or surrounded by a soft cushion while under light anesthesia or sedation. ESWL is not a ureteroscopic approach. ESWL is not done while the patient is undergoing a percutaneous nephrolithotomy.

The nurse notes that a patient who is retaining fluid had a 1-kg weight gain. The nurse knows that this is equivalent to about how many mL?

1,000 mL

A nurse is providing breast cancer education at a community facility. The American Cancer Society recommends that women get mammograms:

yearly starting at age 40.

A client is being discharged to home following a modified radical mastectomy. The nurse is providing discharge instructions and making arrangements for home care. Which of the following interventions will be included in her instructions? Select all that apply.

• Advise the client that blood pressure measurements, injections, blood donations, and IV infusions are lifelong restrictions on the side of the mastectomy. • Recommend wearing gloves while doing yard or housework.

The nurse is caring for a client scheduled for a transrectal ultrasonography. Which instruction is essential in obtaining an accurate test?

Administer an enema prior to arrival.

The client with chronic renal failure complains of intense itching. Which assessment finding would indicate the need for further nursing education?

Brief, hot daily showers

A patient admitted with electrolyte imbalance has carpopedal spasm, ECG changes, and a positive Chvostek sign. What deficit does the nurse suspect the patient has?

Calcium (Hypocalcemia)

When describing the process of fertilization, the nurse would explain that it normally occurs in which structure?

Fallopian tube

Which of the following statements when made by a patient following a laparoscopic procedure requires immediate follow-up by the nurse?

"The area around my navel is tender."

A client is prescribed dutasteride (Avodart) as part of treatment for benign prostatic hyperplasia (BPH). When teaching the client about this drug, the nurse integrates knowledge of the drug's action. Which statement best reflects this?

"The drug interferes with testosterone, which causes the prostate to shrink in size."

An investment banker with chronic renal failure informs the nurse of the choice for continuous cyclic peritoneal dialysis. Which is the best response by the nurse?

"This type of dialysis will provide more independence."

Several male clients are scheduled to come to the clinic for an annual physical examination. The nurse would expect to prepare which client for a digital rectal examination (DRE)?

A 52-year-old man in good health

Which of the following may result if prostate cancer invades the urethra or bladder?

Hematuria

Which of the following occurs late in chronic glomerulonephritis?

Peripheral neuropathy

A nurse who works in a gynecologist's office frequently cares for patients who are diagnosed with vulvovaginal candidiasis. The nurse should teach the patients how to manage and treat the most common symptom of:

Vulvar pruritus. Vulvar pruritus is the chief complaint of those diagnosed with candidiasis.

The nurse recognizes that urinalysis results that most likely indicate a urinary tract infection include:

WBC 50 Increased white blood cell occurs in all clients with a UTI and indicates an infectious process is occurring.

A client is admitted for treatment of chronic renal failure (CRF). The nurse knows that this disorder increases the client's risk of:

Water and sodium retention secondary to a severe decrease in the glomerular filtration rate

A nurse cares for an acutely ill client. The nurse understands that the most accurate indicator of fluid loss or gain in an acutely ill client is:

Weight

A client who has had a left total mastectomy is about to view her surgical site for the first time. Which of the following would be most appropriate for the nurse to say?

"Do you feel like you're ready to look at your incision now?"

A client with end-stage renal disease is scheduled to undergo a kidney transplant using a sibling donated kidney. The client asks if immunosuppressive drugs can be avoided. Which is the best response by the nurse?

"Even a perfect match does not guarantee organ success."

A client has had a right modified radical mastectomy and axillary lymph node dissection. The nurse is teaching the client about measures to reduce the risk of complications. The client demonstrates understanding of the instructions when she states which of the following?

"I need to use an electric shaver when shaving my right armpit."

Which statement demonstrates that a client has understood instructions about breast self-examination?

"I need to use the pads of three fingers to apply pressure when checking each breast."

A nurse is teaching a client who suspects that she has a lump in her breast. The nurse instructs the client that a diagnosis of breast cancer is confirmed by: a) mammography. b) breast self-examination. c) fine needle aspiration. d) chest X-ray.

fine needle aspiration. Correct Explanation: Fine needle aspiration and biopsy provide cells for histologic examination to confirm a diagnosis of cancer. A breast self-examination, if done regularly, is the most reliable method for detecting breast lumps early. Mammography is used to detect tumors that are too small to palpate. Chest X-rays can be used to pinpoint rib metastasis.

A nurse is speaking to a group of women about early detection of breast cancer. The average age of the women in the group is 47. Following the American Cancer Society guidelines, the nurse should recommend that the women

have a mammogram annually.

A client is experiencing symptoms associated with menopause. What is a likely recommendation to increase the client's interest in sexual activity?

low-dose androgens Low-dose androgens are added to the hormone replacement regimen to restore an interest in sexual activity.

Your patient is receiving adjuvant chemotherapy for breast cancer. Which of the following is most likely her node status and tumor size?

node negative, tumor size 1.2 cm

Your patient is receiving adjuvant chemotherapy for breast cancer. Which of the following is most likely her node status and tumor size? a) node negative, tumor size 0.3 cm b) node negative, tumor size 1.2 cm c) node negative, tumor size 0.2 cm d) node negative, tumor size 0.5 cm

node negative, tumor size 1.2 cm Explanation: Adjuvant chemotherapy is recommended for patients who have positive lymph nodes or who have invasive tumors greater that 1 cm in size, regardless of nodal status.

A nurse is providing postprocedure care for a client who underwent percutaneous lithotripsy. In this procedure, an ultrasonic probe inserted through a nephrostomy tube into the renal pelvis generates ultra-high-frequency sound waves to shatter renal calculi. The nurse should instruct the client to:

notify the physician about cloudy or foul-smelling urine. The nurse should instruct the client to report the presence of foul-smelling or cloudy urine to the physician. Unless contraindicated, the client should be instructed to drink large quantities of fluid each day to flush the kidneys. Sandlike debris is normal because of residual stone products. Hematuria is common after lithotripsy.

The nurse cares for a client with end-stage kidney disease (ESKD). Which acid-base imbalance is associated with this disorder?

pH: 7.20, PaCO2: 36, HCO3: 14

A nurse is teaching a female client with a history of multiple urinary tract infections (UTIs). Which statement indicates the client understands the teaching about preventing UTIs?

"I should take at least 1,000 mg of vitamin C each day." The client demonstrates understanding of teaching when she states that she should take vitamin C each day. Increasing vitamin C intake to at least 1,000 mg per day helps acidify the urine, decreasing the amount of bacteria that can grow. The client should wipe from front to back to avoid introducing bacteria from the anal area into the urethra. The client should shower, not bathe, to minimize the amount of bacteria that can enter the urethra. The client should increase her fluid intake, and void every 2 to 3 hours and completely empty her bladder. Holding urine in the bladder can cause the bladder to become distended, which places the client at further risk for UTI.

The nurse is preparing the client for magnetic resonance imaging (MRI) of the kidney. Which statement by the client requires action by the nurse?

"I took my blood pressure medication with my morning coffee an hour ago." The client should not eat for at least 1 hour before an MRI. Alcohol, caffeine-containing beverages, and smoking should be avoided for at least 2 hours before an MRI. The client can take his or her usual medications except for iron supplements prior to the procedure.

Cone biopsy is a term used to describe a procedure in which cervical tissue is removed as result of detection of abnormal cells. Which of the following statements made by the patient demonstrates that the patient undergoing a cone biopsy understands the discharge instructions?

"I will avoid having sexual relations until I see the doctor again."

Which statement by the client who is performing self-catheterization indicates a need for further teaching?

"I will need a sterile catheter kit each time I self-catheterize." Clients who self-catheterize use clean technique in the home setting.

An elderly client, who can void only while standing and pushing upward on the vagina, is ordered a pessary. Which comment from the client indicates a need for further teaching about this device?

"I will remove and clean it every day." Pessaries should be removed, cleaned, and replaced periodically (at least every 2 months). If the client is unable to manage the pessary, then follow-up appointments should be made with the practitioner. Ill-fitting pessaries can cause irritation and erosion of tissue resulting in irritation, bleeding, or infection. The placement of the pessary should support the uterus in the pelvis and lessen urinary and pelvic symptoms.

A client undergoes extracorporeal shock wave lithotripsy. Before discharge, the nurse should provide which instruction?

"Increase your fluid intake to 2 to 3 L per day." The nurse should instruct the client to increase his fluid intake. Increasing fluid intake flushes the renal calculi fragments through — and prevents obstruction of — the urinary system. Measuring temperature every 4 hours isn't needed. Lithotripsy doesn't require an incision. Hematuria may occur for a few hours after lithotripsy but should then disappear.

The nurse is preparing a client for a pelvic examination. The client asks the nurse, "Why do I need to urinate and empty my bladder?" Which response by the nurse would be most appropriate?

"It helps make you more comfortable and the exam easier."

After teaching a group of students about how to perform peritoneal dialysis, which statement would indicate to the instructor that the students need additional teaching?

"It is appropriate to warm the dialysate in a microwave."

A client with urinary incontinence asks the nurse for suggestions about managing this condition. Which suggestion would be most appropriate?

"Make sure to eat enough fiber to prevent constipation." Suggestions to manage urinary incontinence include avoiding constipation such as eating adequate fiber and drinking adequate amounts of fluid. Scented powders, lotions, or sprays should be avoided because they can intensify the urine odor, irritate the skin, or cause a skin infection. Stimulants such as caffeine, alcohol, and aspartame should be avoided. The client should void regularly, approximately every 2 to 3 hours to ensure bladder emptying.

The nurse is completing community education when asked by a client without health insurance why a mammogram is needed if the women are completing breast self-examinations at home. The nurse is most correct to respond stating which of the following?

"Mammograms can detect cysts or tumors too small to palpate."

A nurse is preparing to obtain a sexual history from a client using the PLISSIT model. Which question would be most appropriate for the nurse to ask first?

"May I ask you some questions about your sexual health?"

A 32-year-old client has been diagnosed with an ovarian cyst and asks the nurse, "Should I be worried about cancer?" Which response by the nurse would be most appropriate?

"Most cysts are benign in younger women, but it's good to have it checked out." The risk of malignancy in postmenopausal women is much greater than in premenopausal women, with almost all pelvic masses in premenopausal women being benign. However, although typically benign, the cyst should be evaluated to exclude ovarian cancer. The most appropriate response would be to provide this information to the client but also reinforce her actions to have the cyst evaluated. Telling the client that the cyst is no cause for concern or that ovarian cysts are rarely malignant ignores the client's feelings and does not address the client's concern. The absence of symptoms does not eliminate the possibility of a malignancy. Often, ovarian cancer is difficult to detect and signs and symptoms are vague and nonspecific.

A client is being treated for trichomoniasis. The client has received instructions about the prescribed drug therapy. The nurse determines that the client needs additional teaching when she states which of the following?

"My partner will not need any treatment." Trichomoniasis is treated with metronidazole. Both partners are treated and receive a one-time loading dose or smaller dose three times a day for a week. The client may experience a transient metallic taste when taking the drug. Client are strongly advised to abstain from alcohol when taking metronidazole because of a possible disulfiram-like reaction.

The nurse is caring for a client who is scheduled for the creation of an ileal conduit. Which statement by the client provides evidence that client teaching was effective?

"My urine will be eliminated through a stoma." An ileal conduit is a non-continent urinary diversion whereby the ureters drain into an isolated section of ileum. A stoma is created at one end of the ileum, exiting through the abdominal wall.

A client who wishes to preserve childbearing asks the nurse to explain how taking oral contraceptives will work in the management of endometriosis. Which is the best response by the nurse?

"Symptoms of endometriosis are increased during normal menstrual cycle." The use of estrogen-progestin contraceptives keeps the client in a non bleeding phase of the menstrual cycle, therefore decreasing ectopic tissue from shedding and causing extra uterine bleeding. Blood that is trapped in the peritoneal cavity causes more pain as adhesions form. Endometriosis is cured by natural or surgical menopause but can be medically managed for periods of time with the use of oral contraceptives.

A nurse is meeting with a woman scheduled to have a modified radical mastectomy to remove an aggressive breast tumor. The woman tells the nurse that she agreed to have the surgery before considering alternative options. Which of the following statements is the nurse's best response?

"Tell me more about your fears and concerns."

A client is scheduled for a transurethral rescection of the prostate (TURP). Which statement demonstrates that the expected outcome of "client demonstrates understanding of the surgical procedure and aftercare" has been met?

"The surgeon is going to insert a scope through my urethra to remove a portion of the gland."

A client with urinary tract infection is prescribed phenazopyridine (Pyridium). Which of the following instructions would the nurse give the client?

"This medication will relieve your pain." Phenazopyridine (Pyridium) is a urinary analgesic agent used for the treatment of burning and pain associated with UTIs.

An 56-year-old menopausal client comes to the clinic reporting painful intercourse. Her pelvic examination reveals vaginal dryness. Which suggestion would be most appropriate?

"Try using a water-soluble lubricant before intercourse."

Which statement by the client with end-stage renal disease indicates teaching by the nurse was effective?

"Ultrafiltration methods take much longer than hemodialysis"

A patient who is scheduled for a gynecologic examination and Pap smear informs the nurse that she just began her menstrual cycle. What is the best response by the nurse?

"We will reschedule your examination when you have finished menstruating."

A client presents at the clinic with reports of urinary retention. What question should the nurse ask to obtain additional information about the client's report?

"When did you last urinate?" The nurse needs to determine the last time the client voided.

A 15-year-old high student reports to the school nurse with complaints of cramping, nausea, diarrhea, and backache. Which of the following assessment questions by the nurse is most appropriate in this situation?

"When was your last period?"

A nurse is assisting the physician conducting a cystogram. The client has an intravenous (IV) infusion of D5W at 40 ml/hr. The physician inserts a urinary catheter into the bladder and instills a total of 350 ml of a contrast agent. The nurse empties 500 ml from the urinary catheter drainage bag at the conclusion of the procedure. How many milliliters does the nurse record as urine?

150 mL The urinary drainage bag contains both the contrast agent and urine at the conclusion of the procedure. Total contents (500 ml) in the drainage bag consist of 350 ml of contrast agent and 150 ml of urine.

In which stage of breast development does breast budding occur? a) 3 b) 2 c) 1 d) 4

2 Correct Explanation: Breast budding occurs in Stage 2 of breast development. Stage 1 describes a prepubertal breast. Stage 3 involves further enlargement of breast tissue and the areola. Stage 4 occurs when the nipple and areola form a secondary mound on top of the breast tissue

Atypical hyperplasia increases a women's risk for breast cancer about how many times compared with that of the general population? a) 4 b) 6 c) 8 d) 2

4 Correct Explanation: Atypical hyperplasia increases a women's risk for breast cancer about four to five times compared with that of the general population.

The nurse is caring for a client with chronic kidney disease. The patient has gained 4 kg in the past 3 days. In milliliters, how much fluid retention does this equal? __________________

4,000 mL

The nurse is discussing mammography with a female patient at the clinic. The patient asks at what age she should begin getting yearly mammograms. What answer should the nurse provide to the patient?

40

The nurse is educating a patient about the best time to perform BSE. When does the nurse inform her is the best time after menses to perform BSE?

5 to 7 days

The nurse is caring for several older clients. For which client would the nurse be especially alert for signs and symptoms of pyelonephritis?

A client with urinary obstruction The client with urinary obstruction is at the highest risk of developing pyelonephritis because a urinary obstruction is the most common cause of pyelonephritis in older adults. Acute glomerulonephritis usually occurs in older adults with preexisting chronic glomerulonephritis. Older clients with acute renal failure or urinary tumor are not at high risk for developing pyelonephritis.

A patient reports to the nurse that she has a sense of pelvic pressure and urinary problems such as incontinence, frequency, and urgency. The problem has gotten much worse since the birth of her third child. What does the nurse suspect the patient is experiencing?

A cystocele Cystocele is a downward displacement of the bladder toward the vaginal orifice (Fig. 57-3) from damage to the anterior vaginal support structures. It usually results from injury and strain during childbirth. Because a cystocele causes the anterior vaginal wall to bulge downward, the patient may report a sense of pelvic pressure and urinary problems such as incontinence, frequency, and urgency. Back pain and pelvic pain may occur as well. The symptoms of rectocele resemble those of cystocele, with one exception: Instead of urinary symptoms, patients may experience rectal pressure. Constipation, uncontrollable gas, and fecal incontinence may occur in patients with complete tears.

During a breast examination, which finding most strongly suggests that a client has breast cancer?

A fixed nodular mass with dimpling of the overlying skin

The nurse observes a client's uric acid level of 9.3 mg/dL. When teaching the client about ways to decrease the uric acid level, which diet would the nurse suggest?

A low-purine diet The nurse would suggest a low-purine diet. Foods to avoid are anchovies, animal organs and sardines. The other options do not lower the uric acids levels.

The nurse is providing shift report related to a client newly received back to the unit from the post anesthesia care unit (PACU). The nurse is stating that the client had breast tissue removed with 7 of 14 lymph nodes, the lining of the chest muscles and pectoralis minor muscle removed. The oncoming nurse documents which procedure completed? a) A modified radical mastectomy b) A radical mastectomy c) A total mastectomy d) A segmental mastectomy

A modified radical mastectomy Explanation: A modified radical mastectomy is a procedure in which the breast, some lymph nodes, the lining over the chest muscles, and the pectoralis minor muscle is removed. A segmental mastectomy is where the tumor and some breast tissue and lymph nodes are removed. A total mastectomy includes only breast tissue. A radical mastectomy includes the breast axillary lymph nodes, and pectoralis major and minor muscles are removed. Sternal lymph nodes may also be removed with this procedure.

A 54-year-old woman presents to her healthcare provider's office where you practice nursing. She is very concerned that she might have breast cancer, especially after caring for her sister that recently died from the disease. Included in your discussion is the primary and most common sign of breast cancer. Which of the following would meet this criterion?

A painless mass in the breast, most often in the upper outer quadrant

A group of students are reviewing material in preparation for a test on the male and female breasts. The students demonstrate understanding of the material when they identify which of the following?

A primary function of the female breast is to produce milk.

A client is scheduled for a renal angiography. Which of the following would be appropriate before the test? Monitor the client for an allergy to iodine contrast material. Evaluate the client for periorbital edema. Assess the client's mental changes. Monitor the client for signs of electrolyte and water imbalance.

A. A renal angiography procedure will be contraindicated if the client is allergic to iodine contrast material. Therefore, it is important for the nurse to monitor the client for an allergy to iodine contrast material. The nurse monitors the client for the signs of electrolyte and water imbalance, mental changes, and periorbital edema at any time regardless of the test being done.

A client has a suspected diagnosis of bladder stones. Stones may form in the bladder or originate in the upper urinary tract and travel to and remain in the bladder. What are some signs and symptoms that this client may be experiencing? Select all that apply.

All choices are true. Symptoms of bladder stone formation include hematuria, suprapubic pain, difficulty starting the urinary stream, symptoms of a bladder infection, and a feeling that the bladder is not completely empty. Some clients may have few or no symptoms.

A patient is being seen in the clinic for possible kidney disease. What major sensitive indicator of kidney disease does the nurse anticipate the patient will be tested for? Creatinine clearance level Serum potassium level Blood urea nitrogen level Uric acid level

A. Creatinine is an endogenous waste product of skeletal muscle that is filtered at the glomerulus, passed through the tubules with minimal change, and excreted in the urine. Hence, creatinine clearance is a good measure of the glomerular filtration rate (GFR), the amount of plasma filtered through the glomeruli per unit of time. Creatinine clearance is the best approximation of renal function. As renal function declines, both creatinine clearance and renal clearance (the ability to excrete solutes) decrease.

A client has a full bladder. Which sound would the nurse expect to hear on percussion? Dullness Tympany Flatness Resonance

A. Dullness on percussion indicates a full bladder; tympany indicates an empty bladder. Resonance is heard over areas that are part air and part solid, such as the lungs. Flatness is heard over very dense tissue, such as the bone or muscle.

When describing the functions of the kidney to a client, which of the following would the nurse include? Control of water balance Regulation of white blood cell production Synthesis of vitamin K Secretion of enzymes

A. Functions of the kidneys include control of water balance and blood pressure, regulation of red blood cell production, synthesis of vitamin D to active form, and secretion of prostaglandins.

The nurse is providing care to a client who has had a renal biopsy. The nurse would need to be alert for signs and symptoms of which of the following? Bleeding Dehydration Infection Allergic reaction

A. Renal biopsy carries the risk of postprocedure bleeding because the kidneys receive up to 25% of the cardiac output each minute. Therefore, the nurse would need to be alert for signs and symptoms of bleeding. Although infection is also a risk, the risk for bleeding is greater. Dehydration and allergic reaction are not associated with a renal biopsy.

The nurse is reviewing the client's urinalysis results. The finding that is most suggestive of dehydration of the client is: a) Specific gravity 1.035 b) Creatinine 0.7 mg/dL c) Bright yellow urine d) Protein 15 mg/dL

A. Specific gravity is reflective of hydration status. A concentrated specific gravity, such as 1.035, is suggestive of dehydration. Bright yellow urine suggests ingestion of mulitiple vitamins. Proteinuria can be benign or be caused by conditions which alter kidney function. Creatinine measures the ability of the kidney to filter the blood. A level of 0.7 is within normal limits.

A client is scheduled for a renal arteriogram. When the nurse checks the chart for allergies to shellfish or iodine, she finds no allergies recorded. The client is unable to provide the information. During the procedure, the nurse should be alert for which finding that may indicate an allergic reaction to the dye used during the arteriogram? Pruritus Unusually smooth skin Hypoventilation Increased alertness

A. The nurse should be alert for pruritus and urticaria, which may indicate a mild anaphylactic reaction to the arteriogram dye. Decreased (not increased) alertness may occur as well as dyspnea (not hypoventilation). Unusually smooth skin isn't a sign of anaphylaxis.

A client develops decreased renal function and requires a change in antibiotic dosage. On which factor should the physician base the dosage change? Creatinine clearance GI absorption rate Liver function studies Therapeutic index

A. The physician should base changes to antibiotic dosages on creatinine clearance test results, which gauge the kidney's glomerular filtration rate; this factor is important because most drugs are excreted at least partially by the kidneys. The GI absorption rate, therapeutic index, and liver function studies don't help determine dosage change in a client with decreased renal function.

The nurse is giving discharge instructions to the client following a bladder ultrasound. Which statement by the client indicates the client understands the instructions? "I can resume my usual activities without restriction." "It is normal for my urine to be blood-tinged." "I should increase my fluid intake for the rest of the day." "If I have difficulty urinating, I should contact my physician."

A. A bladder ultrasound is a non-invasive procedure. The client can resume usual activities without restriction.

The nurse is providing instructions to the client prior to an intravenous pyelogram. Which statement by the client indicates teaching was effective? "I will feel a warm sensation as the dye is injected." "I will need to drink all of the dye as quickly as possible." "I should remove all jewelry before the test." "I should let the staff know if I feel claustrophobic."

A. A contrast agent is injected into the client for an intravenous pyelogram. The client may experience a feeling of warmth, flushing of the face, or taste a seafood flavor as the contrast infuses. Jewelry does not need to be removed before the procedure. Claustrophobia is not expected.

When the bladder contains 400 to 500 mL of urine, this is referred to as functional capacity. anuria. renal clearance. specific gravity.

A. A marked sense of fullness and discomfort with a strong desire to void usually occurs when the bladder contains 400 to 500 mL of urine, referred to as the "functional capacity." Anuria is a total urine output less than 50 mL in 24 hours. Specific gravity reflects the weight of particles dissolved in the urine. Renal clearance refers to the ability of the kidneys to clear solutes from the plasma.

The nurse is preparing a client for a nuclear scan of the kidneys. Following the procedure, the nurse instructs the client to drink liberal amounts of fluids. maintain bed rest for 2 hours. notify the health care team if bloody urine is noted. carefully handle urine because it is radioactive.

A. After the procedure is completed, the client is encouraged to drink fluids to promote excretion of the radioisotope by the kidneys. The remaining instructions are not associated with a nuclear scan.

Following a renal biopsy, a client reports severe pain in the back, the arms, and the shoulders. Which intervention should be offered by the nurse? Provide analgesics to the client. Distract the client's attention from the pain. Assess the patient's back and shoulder areas for signs of internal bleeding. Enable the client to sit up and ambulate.

C.

A client with a history of bladder retention hasn't voided for 8 hours. A nurse concerned that the client is retaining urine notifies the physician. He orders a bladder ultrasonic scan and placement of an indwelling catheter if the residual urine is greater than 350 mL. The nurse knows that using the bladder ultrasonic scan to measure residual urine instead of placing a straight catheter reduces the risk of: microorganism transfer. incorrect urine output values. client discomfort. prostate irritation.

A. Bladder ultrasonic scanning, a noninvasive way of calculating the amount of urine in the bladder, reduces the risk of transferring microorganisms into the bladder. Use of a straight catheter to measure residual urine increases the transfer of microorganisms into the bladder, and increases, rather than reduces, client discomfort. A bladder ultrasonic scan doesn't reduce the risk of prostate irritation or incorrect urine output values.

A nurse is reviewing the laboratory test results of a client with renal disease. Which of the following would the nurse expect to find? Increased serum creatinine Increased serum albumin Decreased potassium Decreased blood urea nitrogen (BUN)

A. In clients with renal disease, the serum creatinine level would be increased. The BUN also would be increased, serum albumin would be decreased, and potassium would likely be increased.

A client presents to the emergency department complaining of a dull, constant ache along the right costovertebral angle along with nausea and vomiting. The most likely cause of the client's symptoms is: renal calculi. interstitial cystitis. an overdistended bladder. acute prostatitis.

A. Renal calculi usually presents as a dull, constant ache at the costovertebral angle. The client may also present with nausea and vomiting, diaphoresis, and pallor. The client with an overdistended bladder and interstitial cystitis presents with dull, continuous pain at the suprapubic area that's intense with voiding. The client also complains of urinary urgency and straining to void. The client with acute prostatitis presents with a feeling of fullness in the perineum and vague back pain, along with frequency, urgency, and dysuria.

Which is an effect of aging on upper and lower urinary tract function? More prone to develop hypernatremia Acid-base balance Increased blood flow to the kidneys Increased glomerular filtration rate

A. The elderly are more prone to develop hypernatremia. These clients typically have a decreased glomerular filtration rate, decreased blood flow to the kidneys, and acid-base imbalances.

A nurse measures a patient's urinary output every 8 hours. The nurse weighs the importance of these results by comparing the normal 24-hour urinary output with the patient's condition and medication. The normal 24-hour output should be: 1 to 2 L/day 2.5 to 3 L/day 3.5 to 4 L/day 0.4 to 0.8 L/day

A. The normal output of urine every 24 hours is 800 to 1,500 mL. Refer to Table 26-1 in the text. The significance of the 24-hour result will depend on the patient's medical condition.

When fluid intake is normal, the specific gravity of urine should be which of the following? 1.010 to 1.025. >1.025. 1.000. <1.010.

A. Urine specific gravity is a measurement of the kidney's ability to concentrate urine. The specific gravity of water is 1.000. A urine specific gravity of <1.010 may indicate overhydration. A urine specific gravity >1.025 may indicate dehydration.

A female client presents to the health clinic for a routine physical examination. The nurse observes that the client's urine is bright yellow. Which question is most appropriate for the nurse to ask the client? "Do you take multiple vitamin preparations?" "Have you had a recent urinary tract infection?" "Do you take phenytoin daily?" "Have you noticed any vaginal bleeding?"

A. Urine that is bright yellow is an anticipated abnormal finding in the client taking a multivitamin preparation. Urine that is orange may be caused by intake of phenytoin or other medications. Orange- to amber-colored urine may also indicate concentrated urine due to dehydration or fever. Urine that is pink to red may indicate lower urinary tract bleeding. Yellow to milky white urine may indicate infection, pyuria, or, in the female client, the use of vaginal creams.

A history of infection specifically caused by group A beta-hemolytic streptococci is associated with which disorder?

Acute glomerulonephritis

A client comes to the emergency department complaining of severe pain in the right flank, nausea, and vomiting. The physician tentatively diagnoses right ureterolithiasis (renal calculi). When planning this client's care, the nurse should assign the highest priority to which nursing diagnosis?

Acute pain Ureterolithiasis typically causes such acute, severe pain that the client can't rest and becomes increasingly anxious. Therefore, the nursing diagnosis of Acute pain takes highest priority. Diagnoses of Risk for infection and Impaired urinary elimination are appropriate when the client's pain is controlled. A diagnosis of Imbalanced nutrition: Less than body requirements isn't pertinent at this time.

The nurse is reviewing the potassium level of a patient with kidney disease. The results of the test are 6.5 mEq/L, and the nurse observes peaked T waves on the ECG. What priority intervention does the nurse anticipate the physician will order to reduce the potassium level?

Administration of sodium polystyrene sulfonate (Kayexalate)

A nurse is reviewing a female patient's history, which includes the following information:

Age at first pregnancy

A nurse is obtaining a 40-year-old male's health history before performing a physical examination. Which of the following information would most likely not be obtained?

Age of first ejaculate

A client with challenging menopausal symptoms has discussed treatment options with the physician and now has some questions for the nurse. The client asks, "What are the potential risks of hormone replacement therapy?" What is the best answer?

All options are correct. In using hormonal replacement therapy, the risks of breast cancer and the seriousness of future myocardial infarction and stroke may outweigh the potential benefit of alleviating symptoms associated with menopause. The Women's Health Initiative study revealed an increase in breast cancer, blood clots, stroke, and heart disease in postmenopausal women taking HRT.

Kara Carpenter is a 54-year-old woman who just had a left radical mastectomy. The nurse caring for her is providing information on complications that may arise due to removing the axillary lymph nodes. Which of the following would not be included?

All would be included in the discussion.

Medical management of BPH includes pharmacologic therapy. Which of the following medications would the nurse expect the health care provider to prescribe for this diagnosis?

Alpha-adrenergic blocker

A patient with chronic kidney failure experiences decreased levels of erythropoietin. What serious complication related to those levels should the nurse assess for when caring for this client?

Anemia

The nurse is obtaining a health history from a 58-year-old client stating that he is having difficulty obtaining an erection during sexual activity. The client asks how an "erectile medication" works and if there are any side effects to the medication. The nurse explains the action of the medication and directions for use and warns of which side effect related to the client's history?

Angina with nitrate use

The nurse is providing information at the local YMCA about screenings for breast and cervical cancer. The nurse should inform young women that they should begin their screenings at what time?

Annual breast and pelvic examinations are important for all women 21 years of age or older and for those who are sexually active, regardless of age.

The nurse is employed in a urologist's office. Which classification of medication is anticipated for clients having difficulty with urinary incontinence?

Anticholinergic Pharmacologic agents that can improve bladder retention, emptying, and control include anticholinergic drugs. In this classification are medications such as Detrol, Ditropan, and Urecholine. Diuretics eliminate fluid from the body but do not affect the muscles of urinary elimination. Anticonvulsant and cholinergic medications also do not directly help with control.

The nurse is obtaining a history from a male client who states having difficulty achieving and sustaining an erection. When reviewing the medication history, which medication classification does the nurse anticipate?

Antihypertensives

An ileal conduit is created for a client after a radical cystectomy. Which of the following would the nurse expect to include in the client's plan of care?

Application of an ostomy pouch An ileal conduit involves care of a urinary stoma, much like that of a fecal stoma, including the application of an ostomy pouch, skin protection, and stoma care. Intermittent catheterizations and irrigations are appropriate for a continent urinary diverse such as a Kock or Indiana pouch. Exercises to promote sphincter control are appropriate for an ureterosigmoidoscopy.

The nurse instructs a female client about contraceptive options. The nurse explains that the intrauterine device (IUD) is a good contraceptive option for women who:

Are in a monogamous relationship.

A client is frustrated and embarrassed by urinary incontinence. Which measure should the nurse include in a bladder retraining program?

Assessing present voiding patterns The guidelines for initiating bladder retraining include assessing the client's present intake patterns, voiding patterns, and reasons for each accidental voiding. Lowering the client's fluid intake won't reduce or prevent incontinence. The client should be encouraged to drink 1.5 to 2 L of water per day. A voiding schedule should be established after assessment.

A female client is diagnosed with breast abscess. She would like to continue to breast-feed her newborn. Which of the following would be most appropriate in this situation?

Assist the client to pump the breasts to remove breast milk.

A 67-year-old client reports to her healthcare provider that she is having difficulty during intercourse, causing her pain and disrupting the intimacy that she and her husband are accustomed to sharing. What might be the cause of her pain?

Atrophy of bulbourethral glands

The most common route of regional spread is to which area?

Axillary lymph nodes

Which of the following is a term used to describe excessive nitrogenous waste in the blood, as seen in acute glomerulonephritis?

Azotemia

The most frequent reason for admission to skilled care facilities includes which of the following? a) Stroke b) Urinary incontinence c) Congestive heart failure d) Myocardial infarction

B

A patient who complains of a dull, continuous pain in the suprapubic area that occurs with and at the end of voiding would most likely be diagnosed with which of the following? a) A kidney stone b) Interstitial cystitis c) Prostatic cancer d) Acute pyelonephritis

B Pain over the suprapubic area is most likely related to the bladder. Pain intensity would increase with fullness. Pain at the end of voiding is one of the symptoms associated with interstitial cystitis.

The nurse is assessing a client at the diagnostic imaging center. For which diagnostic test would the client assess for an allergy to shellfish? a) Bladder ultrasonography b) Computed tomography with contrast c) Cystoscopy d) Radiography

B The nurse is correct to assess for an allergy to shellfish most times when a contrast medium is ordered. The other options do not necessarily have a contrast medium.

A 30-year-old male patient presents to the clinic for an employment physical. The nurse notes protein in the patient's urine. The nurse understands that transient proteinuria can be caused by which of the following? Select all that apply. a) NSAIDs b) Strenuous exercise c) Prolonged standing d) Diabetes mellitus e) Fever

B, C, E Proteinuria may be a benign finding, or it may signify serious disease. Common benign causes of transient proteinuria are fever, strenuous exercise, and prolonged standing. Causes of persistent proteinuria include glomerular diseases, malignancies, collagen diseases, diabetes, preeclampsia, hypothyroidism, heart failure, exposure to heavy metals, and use of medications, such as drugs, NSAIDs, and angiotensin-converting enzyme (ACE) inhibitors.

A client is experiencing some secretion abnormalities, for which diagnostics are being performed. Which substance is typically reabsorbed and not secreted in urine? chloride glucose potassium creatinine

B. Amino acids and glucose typically are reabsorbed and not excreted in the urine. The filtrate that is secreted as urine usually contains water, sodium, chloride, bicarbonate, potassium, urea, creatinine, and uric acid.

A client has undergone diagnostic testing that involved the insertion of a lighted tube with a telescopic lens. The nurse identifies this test as which of the following? a) Excretory urogram b) Cystoscopy c) Intravenous pyelography d) Renal angiography

B. Cystoscopy is the visual examination of the inside of the bladder using an instrument called a cystoscope, a lighted tube with a telescopic lens. Renal angiography involves the passage of a catheter up the femoral artery into the aorta to the level of the renal vessels. Intravenous pyelography or excretory urography is a radiologic study that involves the use of a contrast medium to evaluate the kidneys' ability to excrete it.

The wall of the bladder is comprised of four layers. Which of the following is the layer responsible for micturition? Inner layer of epithelium Detrusor muscle Submucosal layer of connective tissue Adventitia (connective tissue)

B. The bladder wall contains four layers. The smooth muscle layer beneath the adventitia is known as the detrusor layer. When this muscle contracts, urine is released from the bladder. When the bladder is relaxed, the muscle fibers are closed and act as a sphincter.

The nephrons are the functional units of the kidney, responsible for the initial formation of urine. The nurse knows that damage to the area of the kidney where the nephrons are located will affect urine formation. Identify that area. Renal papilla Renal cortex Renal medulla Renal pelvis

B. The majority of nephrons (80% to 85%) are located in the renal cortex. The remaining 15% to 20% are located deeper in the cortex.

The nurse is completing a routine urinalysis using a dipstick. The test reveals an increased specific gravity. The nurse should suspect which condition? Glomerulonephritis Decreased fluid intake Increased fluid intake Diabetes insipidus

B. When fluid intake decreases, specific gravity normally increases. With high fluid intake, specific gravity decreases. Disorders or conditions that cause decreased urine-specific gravity include diabetes insipidus, glomerulonephritis, and severe renal damage. Disorders that can cause increased specific gravity include diabetes, nephritis, and fluid deficit.

A 24-hour urine collection is scheduled to begin at 8:00 am. When should the nurse initiate the procedure? 6 hours after the urine is discarded After discarding the 8:00 am specimen With the first specimen voided after 8:00 am At 8:00 am, with or without a specimen

B. A 24-hour collection of urine is the primary test of renal clearance used to evaluate how well the kidney performs this important excretory function. The client is initially instructed to void and discard the urine. The collection bottle is marked with the time the client voided. Thereafter, all the urine is collected for the entire 24 hours. The last urine is voided at the same time the test originally began.

Following a voiding cystogram, the client has a nursing diagnosis of risk for infection related to the introduction of bacterial following manipulation of the urinary tract. An appropriate nursing intervention for the client is to: Strain all urine for 48 hours. Encourage high fluid intake. Monitor for hematuria. Apply moist heat to the flank area.

B. A voiding cystogram involves the insertion of a urinary catheter, which can result in the introduction of microorganism into the urinary tract. Fluid intake is encouraged to flush the urinary tract and promote removal of microorganisms. Monitoring for hematuria, applying heat, and straining urine do not address the nursing diagnosis of risk for infection.

The nurse is caring for a client who has presented to the walk-in clinic. The client verbalizes pain on urination, feelings of fatigue, and diffuse back pain. When completing a head-to-toe assessment, at which specific location would the nurse assess the client's kidneys for tenderness? Around the umbilicus The costovertebral angle Above the symphysis pubis The upper abdominal quadrants on the left and right side

B. The nurse is correct to assess the kidneys for tenderness at the costovertebral angle. The other options are incorrect.

The nurse is caring for a client who is describing urinary symptoms of needing to go to the bathroom with little notice. When the nurse is documenting these symptoms, which medical term will the nurse document? Urinary incontinence Urinary urgency Urinary stasis Urinary frequency

B. The nurse would document urinary urgency. Urinary frequency is urinating more frequently than normal often times due to inadequate emptying of the bladder. Urinary incontinence is the involuntary loss of urine. Urinary stasis is a stoppage or diminution of flow.

The nurse is caring for a client prescribed gentamicin 110 mg every 8 hours for 10 days. Which laboratory study is anticipated to monitor medication side effects?

BUN and serum creatinine the client who is on a therapeutic regimen of gentamicin is ordered laboratory studies of a BUN and serum creatinine to monitor for signs of nephrotoxicity related to medication therapy. Nephrotoxicity from the use of an aminoglycoside is reversible if the medication is discontinued. The other laboratory studies do not focus on nephrotoxicity.

Which of the following is the most effective intravesical agent for recurrent bladder cancer?

Bacillus Calmette-Guérin (BCG) BCG is now considered the most effective intravesical agent for recurrent bladder cancer, especially superficial transitional cell carcinoma, because it is an immunotherapeutic agent that enhances the body's immune response to cancer. Chemotherapy with a combination of methotrexate, 5-FU, vinblastine, doxorubicin (Adriamycin), and cisplatin has been effective in producing partial remission of transitional cell carcinoma of the bladder in some patients.

A woman visits her primary health care provider with a complaint of pain and swelling in the vaginal area. The pain is present when she sits and walks; intercourse is painful. The nurse prepares the patient for an examination. The nurse and health care provider suspect that the patient may have an inflammation or infection of the:

Bartholin glands.

A client has undergone a total abdominal hysterectomy and bilateral salpingo-oopherectomy as treatment for endometrial cancer. When providing postoperative care to this client the nurse would be alert for signs and symptoms of which of the following?

Bladder dysfunction After a total abdominal hysterectomy and bilateral salpingo-oopherectomy, the client is at risk for several complications, especially bladder dysfunction because the surgical site is close to the bladder. Leukopenia and neurotoxicity are adverse effects of chemotherapy agents such as paclitaxel and carboplatin used to treat ovarian cancer. Deep vein thrombosis, not clotting deficiencies are a potential complication after this type of surgery.

Behavioral interventions for urinary incontinence can be coordinated by a nurse. A comprehensive program that incorporates timed voiding and urinary urge inhibition is referred to as what?

Bladder retraining Bladder retraining includes a timed voiding schedule and urinary urge inhibition exercises. These exercises involve delaying voiding to help the patient stay dry for a set period of time. When one time interval is reached, another is set. The time is usually increased by 10 to 15 minutes, until an acceptable voiding interval is achieved.

Fertility studies for men include a semen analysis to determine sperm count, sperm motility, and abnormal sperm. Which of the following tests can also be performed to determine infertility?

Blood test for LH

A client with chronic renal failure (CRF) is admitted to the urology unit. Which diagnostic test results are consistent with CRF?

Blood urea nitrogen (BUN) 100 mg/dL and serum creatinine 6.5 mg/dL

The presence of prerenal azotemia is a probable indicator for hospitalization for CAP. Which of the following is an initial laboratory result that would alert a nurse to this condition?

Blood urea nitrogen (BUN)-to-creatinine ration (BUN:Cr) >20

A nurse is describing the advantages and disadvantages associated with tamoxifen therapy as a means of breast cancer prevention. The nurse would identify that the drug has a beneficial effect on which of the following?

Bone mineral density

During consultations with the oncologist, a client with cervical cancer is informed that radioactive materials will be inserted in the area of the tumor. She has to undergo these sessions over a period of 3 weeks. Which method of cancer treatment is being described?

Brachytherapy Internal radiation, or brachytherapy, delivers a dose of radiation to a localized area inside the body through the use of an implant. The implant may be applied by a needle, seed, bead, or catheter, or can be given orally. Antineoplastic agents used in chemotherapy interfere with the cellular function of the cancer cells and cause cell death. Needle aspiration biopsy involves aspirating tissue fragments through a needle guided into the cancer cells.

Which of the following is considered a diagnostic tool for breast cancer? a) Clinical breast exam b) Ultrasonography c) Mammography d) Breast biopsy

Breast biopsy Correct Explanation: Breast biopsy is a tool used to diagnose breast cancer. Ultrasonography, mammography, and clinical breast exam are all tools used to screen for breast cancer.

The nurse is educating a group of women at the YMCA about breast cancer. What does the nurse understand is the current trend that should be focused on rather than BSE?

Breast self-awareness

After teaching a group of students about the signs and symptoms of breast cancer, the instructor determines that additional teaching is needed when the group identifies which of the following?

Breast symmetry

A nurse is conducting a health history on a patient who is seeing her health care provider for symptoms consistent with a UTI. The nurse understands that the most common route of infection is which of the following?

By ascending infection (transurethral) The most common route of infection is transurethral, in which bacteria colonize the periurethral area and enter the bladder by means of the urethra.

The client is admitted to the hospital with a diagnosis of acute pyelonephritis. Which clinical manifestations would the nurse expect to find? a) Pain after voiding b) Suprapubic pain c) Costovertebal angle tenderness d) Perineal pain

C Acute pyelonephritiis is characterized by costovertebal angle tenderness. Suprapubic pain is suggestive of bladder distention or infection. Urethral trauma and irritation of the bladder neck can cause pain after voiding. Perineal pain is experienced by male clients with prostate cancer or prostatitis.

Renal function results may be within normal limits until the GFR is reduced to less than which percentage of normal? a) 20% b) 40% c) 50% d) 30%

C Renal function test results may be within normal limits until the GFR is reduced to less than 50% of normal. Renal function can be assessed most accurately if several tests are performed and their results are analyzed together. Common tests of renal function include renal concentration tests, creatinine clearance, and serum creatinine and BUN (nitrogenous end product of protein metabolism) levels.

The nurse is aware, when caring for patients with renal disease, that which substance made in the glomeruli directly controls blood pressure? Vasopressin Cortisol Renin Albumin

C. Renin is directly involved in the control of arterial blood pressure. It is also essential for the proper functioning of the glomerulus and management of the body's renin-angiotensin system (RAS).

The nurse is caring for a client scheduled for urodynamic testing. Following the procedure, which information does the nurse provide to the client? "You will be sent home with a urinary catheter." "You may resume consuming caffeinated, carbonated, and alcoholic beverages." "Contact the primary provider if you experience fever, chills, or lower back pain." "You can stop taking the prescribed antibiotic."

C. The client must be made aware of the signs of a urinary tract infection after the procedure. The client should contact the primary provider if fever, chills, lower back pain, or continued dysuria and hematuria occur. The client will have catheters placed during the procedure but will not be sent home with one. The client should be told to avoid caffeinated, carbonated, and alcoholic beverages after the procedure because these can further irritate the bladder. These symptoms usually decrease or subside by the day after the procedure. If the client received an antibiotic medication before the procedure, they should be told to continue taking the complete course of medication after the procedure. This is a measure to prevent infection.

The health care provider ordered four tests of renal function for a patient suspected of having renal disease. Which of the four is the most sensitive indicator? Uric acid level Blood urea nitrogen (BUN) Creatinine clearance level BUN to creatinine ratio

C. The creatinine clearance measures the volume of blood cleared of endogenous creatinine in 1 minute. This serves as a measure of the glomerular filtration rate. Therefore the creatinine clearance test is a sensitive indicator of renal disease progression.

Approximately what percentage of blood passing through the glomeruli is filtered into the nephron? a) 10 b) 30 c) 20 d) 40

C. Under normal conditions, about 20% of the blood passing through the glomeruli is filtered into the nephron, amounting to about 180 liters per day of filtrate.

A client is undergoing diagnostics due to a significant drop in renal output. The physician has scheduled an angiography. What postprocedural assessment will the nurse perform on the client? Palpate pedal pulses. Monitor site condition. All options are correct. Monitor hypersensitivity response.

C. After the procedure, the physician applies a pressure dressing to the femoral area, which remains in place for several hours. The nurse palpates the pulses in the legs and feet at least every 1 to 2 hours for signs of arterial occlusion. Monitoring the pressure dressing is important to note frank bleeding or hematoma formation. If either condition occurs, the nurse immediately notifies the physician. Another important assessment is for hypersensitivity responses to contrast material. The client remains on bed rest for 4 to 8 hours. The nurse also monitors and documents intake and output.

-Which of the following hormones is secreted by the juxtaglomerular apparatus? Calcitonin Antidiuretic hormone (ADH) Renin Aldosterone

C. Renin is a hormone directly involved in the control of arterial blood pressure; it is essential for proper functioning of the glomerulus. ADH, also known as vasopressin, plays a key role in the regulation of extracellular fluid by excreting or retaining water. Calcitonin regulates calcium and phosphorous metabolism.

The nurse is caring for a client who is brought to the emergency department after being found unconscious outside in hot weather. Dehydration is suspected. Baseline lab work including a urine specific gravity is ordered. Which relation between the client's symptoms and urine specific gravity is anticipated? The specific gravity will be low The specific gravity will equal to one The specific gravity will be high. The specific gravity will be inversely proportional

C. The nurse assesses all of the data to make an informed decision on client status. On a hot day, the client found outside will be perspiring. When dehydration occurs, a client will have low urine output and increased specific gravity of urine. Normal specific gravity is inversely proportional. The density of distilled water is one. A low specific gravity is noted in a client with high fluid intake and who is not losing systemic fluid.

A group of students is reviewing for a test on the urinary and renal system. The students demonstrate understanding of the information when they identify which of the following as part of the upper urinary tract? Urethra Bladder Ureters Pelvic floor muscles

C. The upper urinary tract is composed of the kidneys, renal pelvis, and ureters. The lower urinary tract consists of the bladder, urethra, and pelvic floor muscles.

The nurse is discussing nutritional needs for a postmenopausal patient. What dietary increase should the nurse recommend to the patient?

Calcium

The nurse is collecting assessment data on a client who is reporting a vaginal discharge that is cottage cheese-like in appearance. Which pathogen is the most likely cause for this symptom?

Candida albicans Candida albicans is a yeast infection that presents with a thick, curdy white discharge. Gonococcus is the organism that causes gonorrhea and presents with a yellow, mucopurulent discharge. Trichomonas vaginalis presents with a foamy, white, foul-smelling discharge and Gardnerella vaginalis is a watery, fishy-smelling discharge.

Over the past 2 months, a client has been receiving treatment for multiple ear infections and tonsillitis. The client reports a curdy white vaginal discharge and burning with urination. What is the most likely cause of these symptoms?

Candida albicans Candida albicans presents with a thick, curdy white discharge, accompanied by a strong odor and burning with urination. Trichomonas vaginalis presents with a foamy, yellow-white discharge, accompanied by a foul odor and severe itching. Gardnerella vaginalis presents with a watery, gray-white discharge, accompanied by a fishy odor and more discharge after intercourse.

A woman comes to the clinic complaining of vaginal itching and a discharge. Inspection reveals a thick curdlike white discharge. The nurse suspects which of the following?

Candida infection

A client develops acute renal failure (ARF) after receiving IV therapy with a nephrotoxic antibiotic. Because the client's 24-hour urine output totals 240 mL, the nurse suspects that the client is at risk for:

Cardiac arrhythmia

The nurse is to check residual urine amounts for a client experiencing urinary retention. Which of the following would be most important?

Catheterize the client immediately after the client voids. To obtain accurate residual volumes, it is important that clients void first and that catheterization occur immediately after the attempt. The nurse should record both the volume voided (even if it is zero) and the volume obtained by catheterization. Intermittent catheterizations are performed based on a schedule, usually 3 to 4 times per day. Residual urine refers to the amount remaining in the bladder after voiding. It is essential that the client voids.

A woman in her late 30s has been having unusually heavy menstrual periods combined with occasional urine and stool leakage over the past few weeks. Upon further enquiry, she reveals that she also has postcoital pain and bleeding. To which diagnosis will the investigation most likely lead?

Cervical cancer The client's symptoms are those of cervical cancer. Symptoms of cervical cancer include abnormal vaginal bleeding and persistent yellowish, blood-tinged, or foul-smelling discharge. Clients may complain of postcoital pain and bleeding, bleeding between menstrual periods, and unusually heavy menstrual periods. If the cancer has progressed into the pelvic wall, the Clients may experience pain in the flank regions of the body.

A 35-year-old African American who is a regular smoker visits the health care unit with sustained elevated blood levels and is diagnosed with essential hypertension. She is also diabetic. Which of the following contraceptive methods is best for this patient?

Cervical cap

Which objective symptom of a UTI is most common in older adults, especially those with dementia?

Change in cognitive functioning The most common objective finding is a change in cognitive functioning, especially in those with dementia, because these clients usually exhibit even more profound cognitive changes with the onset of a UTI. Incontinence, hematuria, and back pain are not the most common presenting objective symptoms.

The nurse working with a client after an ileal conduit notices that the pouching system is leaking small amounts of urine. What is the appropriate nursing intervention?

Change the wafer and pouch. Whenever a leaking pouching system is noted, the nurse should change the wafer and pouch. Attempting to secure or patch the leak with tape and/or barrier paste can trap urine under the barrier or faceplate, which will compromise peristomal skin integrity. Emptying the pouch will not rectify the leaking.

When preparing a client for hemodialysis, which of the following would be most important for the nurse to do?

Check for thrill or bruit over the access site

A 33-year-old female patient with three children has had a follow-up mammogram following an abnormal BSE. Mammogram findings reveal an incidental microscopic abnormal tissue growth in the left breast lobules. The physician orders tamoxifen (Soltamox) for the patient. The nurse understands that the physician is implementing which of the following primary prevention modalities to treat this patient?

Chemoprevention

A 33-year-old female patient with three children has had a follow-up mammogram following an abnormal BSE. Mammogram findings reveal an incidental microscopic abnormal tissue growth in the left breast lobules. The physician orders tamoxifen (Soltamox) for the patient. The nurse understands that the physician is implementing which of the following primary prevention modalities to treat this patient? a) Chemoprevention b) Radiation therapy c) Prophylactic mastectomy d) Long-term surveillance

Chemoprevention Correct Explanation: Chemoprevention is a primary prevention modality that aims at preventing the disease before it starts.

Which of the following is a primary prevention modality that aims at preventing the disease before it starts? a) Prophylactic mastectomy b) Chemoprevention

Chemoprevention Explanation: Chemoprevention is a primary prevention modality that aims at preventing the disease before it starts

A patient has been diagnosed with a UTI and is prescribed an antibiotic. What first-line fluoroquinolone antibacterial agent for UTIs has been found to be significantly effective?

Cipro Ciprofloxacin (Cipro) is a fluoroquinolone used to treat UTIs. Co-trimoxazole (Bactrim, Septra) is a trimethoprim-sulfamethoxazole combination medication. Nitrofurantoin (Macrodantin, Furadantin) is an anti-infective urinary tract medication.

The nurse is educating a client who is required to restrict potassium intake. What foods would the nurse suggest the client eliminate that are rich in potassium?

Citrus fruits

As the nurse comes from morning report, she is instructed to use a bladder scanner on a client following a client's attempt at urination. The client is able to void 300 mL. The client denies any pain on urination. The nurse scans 250 mL of remaining urine in the bladder. Which entry is most correct when documenting the intervention?

Client voided 300 mL with 250 mL residual volume When documenting the results of using a bladder scanner, it is best to note the amount voided and then the residual urine remaining in the bladder. This documentation enables the analysis of the client's ability to empty the bladder.

A female client who is diagnosed with a malignant tumor in her bladder is advised to undergo cystectomy followed by a urinary diversion procedure. Which of the following would be most important for the nurse to assess preoperatively?

Client's manual dexterity and vision It is essential to assess manual dexterity, vision, and level of understanding of a client who undergoes a urinary diversion procedure, because this information will determine the client's ability to manage stoma care and self-catheterization following the urinary diversion procedure. The client's history of allergy to iodine and seafood, dietary habits related to high cholesterol intake, and menstrual history are not important factors for this situation.

A patient comes to the clinic suspecting a possible UTI. What symptoms of a UTI would the nurse recognize from the assessment data gathered?

Cloudy urine The nurse should observe for signs and symptoms of UTI: cloudy malodorous urine, hematuria, fever, chills, anorexia, and malaise.

The nurse is encouraging the client with recurrent urinary tract infections to increase his fluid intake to 8 large glasses of fluids daily. The client states he frequently drinks water and all of the following. Which of the following would the nurse discourage for this client?

Coffee in the morning The nurse would discourage drinking coffee. Coffee, tea, alcohol, and colas are urinary tract irritants. Fruit juice, milk, and ginger ale are appropriate for drinking and countered toward the daily fluid total.

A group of students is reviewing information about breast cancer and metastasis in preparation for an examination. The students demonstrate the need for additional study when they identify which of the following as a common site for metastasis? a) colon b) liver

Colon Explanation: Breast cancer typically does not metastasize to the colon. The four major sites of metastasis include the skeletal and pulmonary systems, brain, and liver.

Which of the following terms is used to describe a procedure in which cervical tissue is removed as result of detection of abnormal cells?

Cone biopsy (Conization)

The nurse is caring for an older client whose chart reveals that the client has a reversible cause of urinary incontinence. The nurse creates a plan of care for which condition?

Constipation Constipation is a reversible cause of urinary incontinence in the older adult. Other reversible causes include acute urinary tract infection, infection elsewhere in the body, decreased fluid intake, a change in a chronic disease pattern, and decreased estrogen levels in menopausal woman. The other answers do not apply.

The nurse working on a woman's cancer treatment floor performs nursing assessments on her assigned patients. It is most important for the nurse to report which of the following assessment findings?

Coolness and mottling of a newly constructed breast site

A client is diagnosed with a palpable mass in her breast. The nurse would anticipate diagnostic testing using which of the following?

Core needle biopsy

A client reports experiencing vulvar pruritus. Which assessment factor may indicate that the client has an infection caused by Candida albicans?

Cottage cheese-like discharge

A 32-year-old client is concerned with the lumps that have developed in her breasts and is fearful of cancer. She reports variability in the size of the lumps. What could be causing her condition?

Cyclical hormonal changes

Which condition is a downward displacement of the bladder toward the vaginal orifice?

Cystocele A cystocele results from damage to the anterior vaginal support structures. A rectocele is a bulging of the rectum into the vagina. Vulvodynia is a painful condition that affects the vulva. A fistula is an abnormal opening between two organs or sites.

The nurse is caring for a patient with a medical history of sickle cell anemia. The nurse understands this predisposes the patient to which of the following possible renal or urologic disorders? a) Kidney stone formation b) Neurogenic bladder c) Proteinuria d) Chronic kidney disease

D A medical history of sickle cell anemia predisposes the patient to the development of chronic kidney disease. The other disorders are not associated with the development of sickle cell anemia.

Which value does the nurse recognize as the best clinical measure of renal function? Volume of urine output Circulating ADH concentration Urine-specific gravity Creatinine clearance

D Creatinine clearance is a good measure of the glomerular filtration rate (GFR), the amount of plasma filtered through the glomeruli per unit of time. Creatinine clearance is the best approximation of renal function. As renal function declines, both creatinine clearance and renal clearance (the ability to excrete solutes) decrease.

The term used to describe painful or difficult urination is which of the following? a) Oliguria b) Anuria c) Nocturia d) Dysuria

D Dysuria refers to painful or difficult urination. Oliguria is urine output less than 0.5 mL/kg/hr. Anuria is used to describe total urine output of less than 50 mL in 24 hours. Nocturia refers to awakening at night to urinate.

A patient has undergone a renal biopsy. After the test, while the patient is resting, the patient complains of severe pain in the back, arms, and shoulders. Which of the following appropriate nursing interventions should be offered by the nurse? Distract the patient's attention from the pain. Enable the patient to sit up and ambulate. Provide analgesics to the patient. Assess the patient's back and shoulder areas for signs of internal bleeding.

D. After a renal biopsy, the patient is on bed rest. It is important to assess the dressing frequently for signs of bleeding and evaluate the type and severity of pain. Severe pain in the back, shoulder, or abdomen may indicate bleeding. In such a case, the nurse should notify the physician about these signs and symptoms. Distracting the patient's attention, helping the patient to sit up or ambulate, and providing analgesics may only aggravate the patient's pain and, therefore, should not be performed by the nurse.

A client reports urinary frequency, urgency, and dysuria. Which of the following would the nurse most likely suspect? Obstruction of the lower urinary tract Nephrotic syndrome Acute renal failure Infection

D. Frequency, urgency, and dysuria are commonly associated with urinary tract infection. Hesitancy and enuresis may indicate an obstruction. Oliguria or anuria and proteinuria might suggest acute renal failure. Nocturia is associated with nephrotic syndrome.

The nurse is caring for a client after a cystoscopic examination. Following the procedure, the nurse informs the client that which effect may occur? Severe abdominal pain Diarrhea Nausea and emesis Blood-tinged urine

D. Postprocedural management is directed at relieving any discomfort resulting from the examination. Some burning upon voiding, blood-tinged urine, and urinary frequency from trauma to the mucous membranes can be expected. Moist heat to the lower abdomen and warm Sitz baths are helpful in relieving pain and relaxing the muscles. Not eating and diarrhea are not expected following a cystoscopic examination. The client should not experience severe abdominal pain.

The nurse is providing care to a client who has had a kidney biopsy. The nurse would need to be alert for signs and symptoms of which of the following? Dehydration Infection Allergic reaction Bleeding

D. Renal biopsy carries the risk of post procedure bleeding, because the kidneys receive up to 25% of the cardiac output each minute. Therefore, the nurse would need to be alert for signs and symptoms of bleeding. Although infection is also a risk, the risk for bleeding is greater. Dehydration and allergic reaction are not associated with a renal biopsy.

Retention of which electrolyte is the most life-threatening effect of renal failure? Phosphorous Calcium Sodium Potassium

D. Retention of potassium is the most life-threatening effect of renal failure.

Which nursing assessment finding indicates the client has not met expected outcomes? a) The client consumes 75% of lunch following an intravenous pyelogram. b) The client has blood-tinged urine following brush biopsy. c) The client reports a pain rating of 3 two hours post-kidney biopsy. d) The client voids 75 cc four hours post cystoscopy.

D. Urinary retention is an undesirable outcome following cystoscopy. A pain rating of 3 is an achieveable and expected outcome following kidney biopsy. Blood-tinged urine is an expected finding following cystoscopy due to trauma of the procedure. A client would be expected to eat and retain a meal following an intravenous pyelogram.

When fluid intake is normal, the specific gravity of urine should be: 1.000 Less than 1.010 Greater than 1.025 1.010 to 1.025

D. Urine-specific gravity is a measurement of the kidneys' ability to concentrate urine. The specific gravity of water is 1.000. A urine-specific gravity less than 1.010 may indicate inadequate fluid intake. A urine-specific gravity greater than 1.025 may indicate overhydration.

Nursing management of the client with a urinary tract infection should include:

Discouraging caffeine intake Strategies for preventing urinary tract infection include proper perineal hygiene, increased fluid intake, avoiding urinary tract irritants (including caffeine), and establishing a frequent voiding regimen.

The term used to describe total urine output less than 0.5 mL/kg/hr is anuria. dysuria. nocturia. oliguria.

D. Oliguria is associated with acute and chronic renal failure. Anuria is used to describe total urine output less than 50 mL in 24 hours. Nocturia refers to awakening at night to urinate. Dysuria refers to painful or difficult urination.

Urine specific gravity is a measurement of the kidney's ability to concentrate and excrete urine. The specific gravity measures urine concentration by measuring the density of urine and comparing it with the density of distilled water. Which is an example of how urine concentration is affected? A. When the kidneys are diseased, the ability to concentrate urine may be impaired and the specific gravity may vary widely. B. A person who has a high fluid intake and who is not losing excessive water from perspiration, diarrhea, or vomiting has scant urine output with a high specific gravity. C. On a hot day, a person who is perspiring profusely and taking little fluid has high urine output with a low specific gravity. D. On a hot day, a person who is perspiring profusely and taking little fluid has low urine output with a high specific gravity.

D. On a hot day, a person who is perspiring profusely and taking little fluid has low urine output with a high specific gravity. A person who has a high fluid intake and who is not losing excessive water from perspiration, diarrhea, or vomiting has copious urine output with a low specific gravity. When the kidneys are diseased, the ability to concentrate urine may be impaired and the specific gravity remains relatively constant.

A patient is scheduled for a test with contrast to determine kidney function. What statement made by the patient should the nurse inform the physician about prior to testing? "I don't like needles." "I take medication to help me sleep at night." "I have had a test similar to this one in the past." "I am allergic to shrimp."

D. The nurse should obtain the patient's allergy history with emphasis on allergy to iodine, shellfish, and other seafood, because many contrast agents contain iodine.

The nurse is instructing a 3-year-old's mother regarding abnormal findings within the urinary system. Which assessment finding would the nurse document as a normal finding for this age group? Hematuria Anuria Dysuria Enuresis

D. The nurse would be most correct to document that enuresis, the involuntary voiding during sleep or commonly called "wetting the bed," is a normal finding in a pediatric client younger than 5 years old. Dysuria (pain on urination), hematuria (red blood cells in urine), and anuria (urine output less than 50 mL/day) are all abnormal findings needing further investigation.

A client has just been diagnosed with breast cancer and has been seen by the physician who has described the various options available to the client. The client tells the nurse, "I just don't know what to do. I'm so frightened. I don't know if surgery is the best thing. But I've heard such horror stories about chemotherapy, too." When developing this client's plan of care which nursing diagnosis would be most appropriate?

Decisional conflict related to the choice of treatment options

A client who suffered hypovolemic shock during a cardiac incident has developed acute renal failure. Which is the best nursing rationale for this complication?

Decrease in the blood flow through the kidneys

Which of the following is an age-related functional change of the female reproductive system?

Decreased ovulation

After teaching a group of students about the types of urinary incontinence and possible causes, the instructor determines that the students have understood the material when they identify which of the following as a cause of stress incontinence?

Decreased pelvic muscle tone due to multiple pregnancies Stress incontinence is due to decreased pelvic muscle tone, which is associated with multiple pregnancies, obstetric injuries, obesity, menopause, or pelvic disease. Transient incontinence is due to increased urine production related to metabolic conditions. Urge incontinence is due to bladder irritation related to urinary tract infections, bladder tumors, radiation therapy, enlarged prostate, or neurologic dysfunction. Overflow incontinence is due to obstruction from fecal impaction or enlarged prostate.

The nurse expects which of the following assessment findings in the client in the diuretic phase of acute renal failure?

Dehydration

A female patient experienced a severe reduction in body fat by extreme caloric restriction and exercise. Which of the following female reproductive abnormalities could she acquire as a result of the reduction in body fat?

Delay or cessation of menstruation

.An older adult client is being evaluated for suspected pyelonephritis and is ordered kidney, ureter, and bladder (KUB) x-ray. The nurse understands the significance of this order is related to which rationale?

Detects calculi, cysts, or tumors Urinary obstruction is the most common cause of pyelonephritis in the older adult. A KUB may reveal obstructions such as calculi, cysts, or tumors. KUB is not indicated for detection of impaired renal function or reveal increased risk for chronic form of the disorder. Urine cultures will reveal causative microorganisms present in the urine.

An older adult client is being evaluated for suspected pyelonephritis and is ordered kidney, ureter, and bladder (KUB) x-ray. The nurse understands the significance of this order is related to which rationale?

Detects calculi, cysts, or tumors Urinary obstruction is the most common cause of pyelonephritis in the older adult. A KUB may reveal obstructions such as calculi, cysts, or tumors. KUB is not indicated for detection of impaired renal function or reveal increased risk for chronic form of the disorder. Urine cultures will reveal causative microorganisms present in the urine.

How can breast cancer prevention programs best serve at-risk women from lower socioeconomic backgrounds?

Develop screening and educational programs.

A nurse who works in a clinic sees many patients with a variety of medical conditions. The nurse understands that a risk factor for UTIs is which of the following?

Diabetes mellitus Increased urinary glucose levels create an infection-prone environment in the urinary tract

The client with chronic renal failure is exhibiting signs of anemia. Which is the best nursing rationale for this symptom?

Diminished erythropoietin production

The nurse is caring for the client following surgery for a urinary diversion. The client refuses to look at the stoma or participate in its care. The nurse formulates a nursing diagnosis of:

Disturbed body image The client is exhibiting defining characteristics of disturbed body image.

The nurse is caring for a group of breast cancer survivors post mastectomy. Which teaching point should the nurse include?

Do not lift objects greater than 15 lb

The nurse caring for a client with a urinary diversion notices mucus around the stents and in the client's urine. Which is the appropriate nursing intervention?

Document presence of mucus in the urine. The nurse should document the presence of mucus in the urine, as this is a normal finding in urinary diversions.

A client is in end-stage chronic renal failure and is being added to the transplant list. The nurse explains to the client how donors are found for clients needing kidneys. Which statement is accurate?

Donors are selected from compatible living donors

Which of the following instructions would be appropriate to include when preparing a woman for an abdominal ultrasound?

Drink at least 1 quart of water an hour before the test.

Which instruction would be included in a teaching plan for a client diagnosed with a urinary tract infection?

Drink liberal amount of fluids. Clients diagnosed with a UTI should drink liberal amounts of fluids. They should void every 2 to 3 hours. Coffee and tea are urinary irritants. The client should shower instead of bathe in a tub because bacteria in the bathwater may enter the urethra.

A 37-year-old client has been diagnosed with breast cancer and is awaiting the cytology results of her biopsy. During your client education session, you discuss the possible types of breast malignancies. Which is the most common type? a) Medullary b) Infiltrating lobular c) Inflammatory d) Ductal

Ductal Explanation: The most common malignancy is ductal carcinoma (80%); followed by infiltrating lobular carcinoma (10%); medullary carcinoma, mucinous carcinoma, and tubular ductal carcinoma; and inflammatory breast cancer, the rarest but most aggressive form of breast cancer. The most common malignancy is ductal carcinoma (80%); inflammatory breast cancer (1% to 3%) is the rarest but most aggressive form of breast cancer. The most common malignancy is ductal carcinoma (80%). The most common malignancy is ductal carcinoma (80%) followed by infiltrating lobular carcinoma (10%).

A nurse is preparing a presentation for a local women's group about benign and malignant breast conditions. Which of the following would the nurse include as an example of a malignant breast condition?

Ductal carcinoma in situ

An adolescent patient comes to the clinic with complaints of "terrible pain" during menstruation. What should the nurse document this subjective data as?

Dysmenorrhea

Menopause marks the end of a woman's reproductive capacity. Which of the following is a common complaint that may be due to a cystocele?

Dyspareunia

Which of the following would be least appropriate to suggest to a client with a urinary diversion to control odor?

Eat plenty of cheese and eggs. To help control odor, the client should use pouches with carbon filters or other odor barriers or add a few drops of liquid deodorizer or diluted white vinegar to the pouch. Foods such as cranberry juice, yogurt or buttermilk may help to decrease odor while foods such as asparagus, cheese, and eggs may impart an odor to the urine.

A male client who is admitted with the diagnosis of urinary calculi complains of excruciating pain. The pain is suspected to be caused by increased pressure in the renal pelvis. Which measure would be most appropriate to provide pain relief?

Encourage frequent ambulation. When a client with urinary calculi complains of excruciating pain, the client should be encouraged to ambulate. This is because the supine position increases colic, while ambulation relieves it. Also, adequate fluid intake should be suggested to promote the passage of stones and to prevent urinary stasis, or the formation of new stones. The client should be encouraged to void when there is a risk of infection related to urinary stasis. The suggestion for restricting sodium intake is offered to a client with chronic glomerulonephritis, not urinary calculi. The nurse should promote deep-breathing exercises to provide relief to a client recovering from surgery who has an ineffective breathing pattern.

A nurse identifies a nursing diagnosis of risk for ineffective breathing pattern related to incisional pain and restricted positioning for a client who has had a nephrectomy. Which of the following would be most appropriate for the nurse to include in the client's plan of care?

Encourage use of incentive spirometer every 2 hours

A client has undergone a transverse rectus abdominis myocutaneous (TRAM) flap procedure for breast reconstruction immediately following a mastectomy. Which of the following would be most appropriate to include in the client's postoperative plan of care? a) Inspecting the breast site for expected mottling b) Maintaining the client in the supine position c) Monitoring the single incisional site at the breast d) Encouraging coughing and deep breathing exercises

Encouraging coughing and deep breathing exercises Explanation: A client who has undergone a TRAM flap procedure has incisions at both the mastectomy and abdominal donor sites. In addition, the breathing and leg exercises are essential because the client is more limited in her activity and is at greater risk for respiratory complications and deep vein thrombosis. Mottling at the newly created breast site must be reported to the surgeon immediately. Elevating the head of the bed 45 degrees and flexing the client's knees help to reduce tension on the abdominal incision during the first postoperative week.

A nurse is caring for a client who had a stroke. Which nursing intervention promotes urinary continence?

Encouraging intake of at least 2 L of fluid daily Encouraging a daily fluid intake of at least 2 L helps fill the client's bladder, thereby promoting bladder retraining by stimulating the urge to void. The nurse shouldn't give the client soda before bedtime; soda acts as a diuretic and may make the client incontinent. The nurse should take the client to the bathroom or offer the bedpan at least every 2 hours throughout the day; twice per day is insufficient. Consultation with a dietitian won't address the problem of urinary incontinence.

Which diagnostic is indicated for postmenopausal bleeding?

Endometrial biopsy

A client is scheduled for an abdominal ultrasound as a follow up to her pelvic examination. Which action by the nurse would be most appropriate?

Ensuring that the client has a full bladder

Which of the following is the descent of the small intestine into the vaginal vault?

Enterocele The descent of the small intestine into the vaginal vault is termed enterocele. A rectocele is the extrusion of the rectum into the posterior vagina. A cystocele is the herniation of the bladder into the anterior vagina. A uterine prolapsed is the downward descent of the uterus into the vagina.

A nurse educator is giving a lecture to a group of boys about normal male reproductive structures. Which of the following male structures collects the spermatocytes and nourishes them until they are able to spontaneously move about?

Epididymis

Which of the following is a cause of a calcium renal stone?

Excessive intake of vitamin D Potential causes of calcium renal stones include excessive intake of vitamin D, hypercalcemia, hyperparathyroidism, excessive intake of milk and alkali, and renal tubular acidosis. Gout is associated with uric acid. Struvite stones are associated with neurogenic bladder and foreign bodies.

Which type of biopsy would the nurse tell the patient is a usual procedure for any palpable breast mass found on mammography?

Excisional

As a grade school nurse, you speak to the sixth grade girls regarding their physical maturation and body changes during puberty. In your student education sessions, you discuss female anatomy and the appropriate names and functions of each structure. Which of the following structures is referred to as the vulva?

External female genitalia

In your role as nursing director at the local free clinic, you spend a significant amount of time instructing young women about their bodies and maintaining sexual health. Instruction regarding conception includes the structures involved and their roles. Within the female internal reproductive structures, where is the egg most commonly fertilized?

Fallopian tubes

The client is asking if there is a pill that can be ordered to control the symptoms of menopause. Which assessment finding is most important in determining nursing care in association with hormone replacement therapy?

Family history of breast cancer The risk of endometrial or breast cancer in women prescribed HRT may outweigh the benefits of relieving symptoms of menopause and preventing kyphosis or hip fractures associated with osteoporosis.

A client with chronic renal failure (CRF) has developed faulty red blood cell (RBC) production. The nurse should monitor this client for:

Fatigue and weakness

The nurse advises a patient scheduled for external beam radiation that side effects are minimal and usually well tolerated. The most common and annoying side effect is:

Fatigue.

A 13-year-old who started her menstrual cycle at age 12 asks the nurse practitioner how frequently her "period" should come. The nurse takes a calendar and has the girl circle the date of her last period, which started on January 7. The nurse then circles when her next period should start based on the average number of days in a normal cycle. What date did the nurse circle?

February 5

Which nursing assessment finding indicates that the client who has undergone renal transplant has not met expected outcomes?

Fever

Which of the following is a characteristic of a breast cancer mass?

Firm, hard, embedded in surrounding tissue

Due to a patient's reluctance to begin hormone therapy, the advance nurse practitioner prescribes venlafaxine (Effexor) to decreases the risk of which of the following?

Hot flashes

A nurse is reviewing a client's history for possible risk factors associated with breast cancer. Which of the following would the nurse identify as increasing the client's risk?

First full-term pregnancy at age 34 years

A nurse is reviewing a client's history for possible risk factors associated with breast cancer. Which of the following would the nurse identify as increasing the client's risk? a) Menopause after age 50 b) Menarche at age 14 c) First full-term pregnancy at age 34.

First full-term pregnancy at age 34 years Correct Explanation: Risk factors associated with breast cancer include menarche before age 12 years, menopause after age 55 years, nulliparity, and late age at first full-term pregnancy.

A 28-year-old woman is learning about breast self-examination. The nurse teaches the woman that the best time of each month to examine her breasts is during the:

First week after menstruation.

Culture of client's vaginal discharge reveals Gardnerella vaginalis. Which of the following would the nurse expect to assess?

Fishy smelling watery discharge Gardnerella vaginalis is associated with a gray white, watery, fishy smelling vaginal discharge. The discharge associated with a Candida infection is curdy white, thick, and strong. Discharge due to trichomonas vaginalis is yellow white, foamy, and foul.

A patient informs the nurse that she believes she has premenstrual syndrome and is having physical symptoms as well as moodiness. What physical symptoms does the nurse recognize are consistent with PMS? (Select all that apply.)

Fluid retention Low back pain Headache

A 30-year-old client whose mother died of breast cancer at age 44 and whose sister has ovarian cancer, is concerned about developing cancer. As a member of the oncology multidisciplinary team, the nurse should suggest that the client ask the physician about which topic?

Genetic counseling

A 30-year-old client whose mother died of breast cancer at age 44 and whose sister has ovarian cancer, is concerned about developing cancer. As a member of the oncology multidisciplinary team, the nurse should suggest that the client ask the physician about which topic? a) Mammogram b) Genetic counseling c) Pap testing d) Contacting the American Cancer Society

Genetic counseling Explanation: The nurse should suggest that the client ask the physician about genetic counseling. Genetic counseling is indicated for those at high risk because of family or personal cancer history. Genetic counseling involves obtaining a detailed medical and three-generational family history; calculating a personalized risk assessment; providing options for prevention, surveillance, and genetic testing; coordinating and interpreting genetic testing; and developing a management plan based on the test results. Mammography will assist with early detection of most breast cancers, but it won't establish a risk assessment and provide options for prevention, surveillance, and genetic testing. Pap testing every 6 months assists in early detection of most cervical cancers, but it won't establish a risk assessment. Contacting the American Cancer Society will provide the client with information about cancer but the organization won't help assess the client's risk for developing cancer.

Which of the following causes should the nurse suspect in a client is diagnosed with intrarenal failure?

Glomerulonephritis

Hematoma and seroma formation are complications of breast surgery. Which of the following is the indicator that should be reported to the surgeon?

Gross swelling

The nurse is obtaining a health history from a client describing urinary complications. Which assessment finding is most suggestive of a malignant tumor of the bladder?

Hematuria The most common first symptom of a malignant tumor is hematuria. Most malignant tumors are vascular; thus, abnormal bleeding can be a first sign of abnormality. The client then has symptoms of incontinence (a later sign), dysuria and frequency.

A client, age 42, visits the gynecologist. After examining the client, the physician suspects cervical cancer. The nurse reviews the client's history for risk factors for this disease. Which history finding is a risk factor for cervical cancer?

Human papillomavirus infection at age 32 Like other viral and bacterial venereal infections, human papillomavirus is a risk factor for cervical cancer. Other risk factors for this disease include frequent sexual intercourse before age 20, multiple sex partners, and multiple pregnancies. A spontaneous abortion and pregnancy complicated by eclampsia aren't risk factors for cervical cancer.

Rejection of a transplanted kidney within 24 hours after transplant is termed

Hyperacute rejection

A client in chronic renal failure becomes confused and complains of abdominal cramping, racing heart rate, and numbness of the extremities. The nurse relates these symptoms to which of the following lab values?

Hyperkalemia

The nurse is caring for a patient in acute renal failure. The nurse should expect hypertonic glucose, insulin infusions, and sodium bicarbonate to be used to treat:

Hyperkalemia

Which metabolic defects are associated with stone formation?

Hyperparathyroidism Metabolic defects such as hyperparathyroidism and hyperuricemia (gout) are associated with stone formation. Hypoparathyroidism, hyperthyroidism, and hypouricemia are not associated with stone formation.

A nurse is reviewing the history of a client who is experiencing difficulty sustaining an erection. Which of the following might the nurse suspect as a possible contributing factor?

Hypertension

The nurse is caring for a patient after kidney surgery. What major danger should the nurse closely monitor for?

Hypovolemic shock caused by hemorrhage

Which type of incontinence refers to the involuntary loss of urine due to extrinsic medical factors, particularly medications?

Iatrogenic Iatrogenic incontinence is the involuntary loss of urine due to extrinsic medical factors, predominantly medications. Reflex incontinence is the involuntary loss of urine due to hyperreflexia in the absence of normal sensations usually associated with voiding. Urge incontinence is the involuntary loss of urine associated with a strong urge to void that cannot be suppressed. Overflow incontinence is the involuntary loss of urine associated with overdistention of the bladder.

The nurse is caring for a client diagnosed with bladder cancer and requiring a cystectomy. The nurse overhears the physician instructing the client on the presence of a stoma with temporary pouch. In gathering information for the client, which urinary diversion would the nurse select?

Ileal conduit When the physician is discussing a stoma, the nurse recognizes that the client will have an ileal conduit which is a cetaceous urinary diversion. Both the Kock Pouch and Indiana Pouch are continent urinary diversions. The ureterosigmoidostomy connects with the rectum for urinary drainage.

The nurse is conducting a community education program on UTIs. The nurse determines that the participants understand the teaching when they identify which factor as contributing to UTIs in older adults?

Immunocompromise Factors that contribute to UTIs in older adults include immunocompromise, cognitive impariment, high incidence of chronic illness, immobility, incomplete emptying of the bladder, obstructed flow of urine, and frequent use of antimicrobial agents.

The nurse is providing education regarding sexually transmitted infections. Which statement regarding herpes virus 2 (herpes genitalis) is accurate?

In pregnant women with active herpes, babies delivered vaginally may become infected with the virus. A cesarean birth may be performed if the virus recurs near the time of delivery. Asexual transmission by contact with wet surfaces or self-transmission (i.e., touching a cold sore and then touching the genital area) can occur. Transmission is possible even when the carrier does not have symptoms.

The nurse is relating health education to male students when asked where sperm is actually made. Which location is most correct?

In the testes within the scrotum

A woman who is 10 weeks pregnant arrives at the emergency department reporting vaginal bleeding and cramping. She states, "I've passed some small clots." Inspection of the perineal pad reveals moderate bleeding and some tissue. The client's cervix is dilated. The nurse interprets these findings as indicative of which of the following?

Incomplete abortion

Which type of voiding dysfunction is seen in clients diagnosed with Parkinson disease?

Incontinence Incontinence is noted in clients diagnosed with Parkinson disease. Urinary retention is associated with spinal cord injury. Urgency is associated with an overactive bladder. Incomplete bladder emptying is associated with diabetes mellitus.

A client has end-stage renal failure. Which of the following should the nurse include when teaching the client about nutrition to limit the effects of azotemia?

Increase carbohydrates and limit protein intake

The nurse is educating a patient with urolithiasis about preventive measures to avoid another occurrence. What should the patient be encouraged to do?

Increase fluid intake so that the patient can excrete 2,500 to 4,000 mL every day, which will help prevent additional stone formation. A patient who has shown a tendency to form stones should drink enough fluid to excrete greater than 2,000 mL (preferably 3,000 to 4,000 mL) of urine every 24 hours (Meschi et al., 2011).

The nurse advises the patient with chronic pyelonephritis that he should:

Increase fluids to 3 to 4 L/24 hours to dilute the urine. Unless contraindicated, fluids should be increased to dilute the urine, decrease burning on urination, and prevent dehydration. A balanced diet would be recommended but there is no need to restrict sodium or calcium.

Patients with urolithiasis need to be encouraged to:

Increase their fluid intake so that they can excrete 2.5 to 4 liters every day. Fluids need to be increased up to 4 L/day to help prevent additional stone formation.

What is a characteristic of the intrarenal category of acute kidney injury (AKI)?

Increased BUN

What is a characteristic of the intrarenal category of acute renal failure?

Increased BUN

The nurse is assessing the breast of a female patient and observes a prominent venous pattern on the left breast. What does the nurse understand that this can be indicative of?

Increased blood supply required by a tumor

Which of the following is true regarding hormonal contraception?

Increases the risk for venous thromboembolism

A female patient comes to the clinic with the complaint that she is having a greenish-colored discharge from the nipple and the breast feels warm to touch. What does the nurse suspect these symptoms may indicate?

Infection

A young client presenting at the health clinic with fever and mucopurulent vaginal drainage is diagnosed with acute pelvic inflammatory disease (PID). Which long-term affect is of greatest concern in the care of this client?

Infertility About one third of all women who are infertile have lost the ability to conceive due to PID. Early treatment of PID prevents the infection from ascending up the reproductive tract and/or from becoming chronic. Pelvic pain is a common symptom associated with PID.

A patient is told that she has a common form of breast cancer where the tumor arises from the duct system and invades the surrounding tissues, often forming a solid irregular mass. What type of cancer does the nurse prepare to discuss with the patient?

Infiltrating ductal carcinoma

A client is administered dialysate solution through an abdominal catheter. The nurse notices that the return flow rate is slow, so the nurse advises the client to move to the other side. However, even after changing the client's position, the nurse does not observe an increase in return flow. Which of the following actions should the nurse perform to help accelerate the return flow rate?

Inform the physician that catheter may need repositioning

The nurse is caring for a client who is beginning doxorubicin (Adriamycin) therapy for breast cancer. When preparing the client for probable side effects, which would the nurse include?

Information regarding wigs from the American Cancer Society

The treatment of choice for a spinal cord-injured patient with impaired bladder emptying would include which of the following?

Intermittent self-catheterization Intermittent self-catheterization is the treatment of choice in patients with spinal cord injury and other neurologic disorders, such as multiple sclerosis (MS), when the ability to empty the bladder is impaired.

A client has a routine Papanicolaou (Pap) test during a yearly gynecologic examination. The result reveals a class V finding. What should the nurse tell the client about this finding?

It calls for a biopsy as soon as possible.

A patient is diagnosed with the most common type of uterine fibroid, an intramural fibroid. The nurse includes which information in teaching the patient about this type of fibroid?

It grows within the wall of the uterine muscle. See Box 33-10 in the text.

The nurse is assessing an older adult female who has not seen her physician in 2 years. The nurse is assisting the patient into a gown and notices that the patient has edema and pitting of the skin on the right breast. What does the nurse understand is the significance of this finding?

It may result from a neoplasm blocking lymphatic drainage, giving the skin an orange-peel appearance, a classic sign of advanced breast cancer.

After undergoing retropubic prostatectomy, a client returns to his room. The client is on nothing-by-mouth status and has an IV infusing in his right forearm at a rate of 100 ml/hour. The client also has an indwelling urinary catheter that's draining light pink urine. While assessing the client, the nurse notes that his urine output is red and has dropped to 15 ml and 10 ml for the last 2 consecutive hours. How can the nurse best explain this drop in urine output?

It's an abnormal finding that requires further assessment. The drop in urine output to less than 30 ml/hour is abnormal and requires further assessment. The reduction in urine output may be caused by an obstruction in the urinary catheter tubing or deficient fluid volume from blood loss. The client's nothing-by-mouth status isn't the cause of the low urine output because the client is receiving I.V. fluid to compensate for the lack of oral intake. Ambulation promotes urination; however, the client should produce at least 30 ml of urine/hour.

A client comes to the emergency department complaining of sudden onset of sharp, severe pain in the lumbar region that radiates around the side and toward the bladder. The client also reports nausea and vomiting and appears pale, diaphoretic, and anxious. The physician tentatively diagnoses renal calculi and orders flat-plate abdominal X-rays. Renal calculi can form anywhere in the urinary tract. What is their most common formation site?

Kidney The most common site of renal calculi formation is the kidney. Calculi may travel down the urinary tract with or without causing damage and lodge anywhere along the tract or may stay within the kidney. The ureter, bladder, and urethra are less common sites of renal calculi formation.

The expert nurse is assisting a novice nurse insert a Foley catheter. The novice nurse has tried unsuccessfully to insert the catheter, and the expert nurse is providing verbal guidance while spreading which area to reveal the urethral opening?

Labia majora and minora

A client is in isolation after receiving an internal radioactive implant to treat cancer. Two hours later, the nurse discovers the implant in the bed linens. What should the nurse do first?

Leave the room and notify the radiation therapy department immediately. If a radioactive implant becomes dislodged, the nurse should immediately leave the room and notify the radiation therapy department. The nurse shouldn't attempt to handle the implant or remain in the room with the implant.

The nurse is caring for a patient in the oliguric phase of acute kidney injury (AKI). What does the nurse know would be the daily urine output?

Less than 400 mL

The nurse in the gynecology clinic is interviewing a patient who informs the nurse that her mother and aunt had carcinoma of the cervix. What does the nurse recognize are two chief symptoms of early carcinoma that the patient should be questioned about?

Leukorrhea and irregular vaginal bleeding or spotting Early cervical cancer rarely produces symptoms. If symptoms are present, they may go unnoticed as a thin, watery vaginal discharge often noticed after intercourse or douching. When symptoms such as discharge, irregular bleeding, or pain or bleeding after sexual intercourse occur, the disease may be advanced.

The nurse is preparing the female client for a pelvic examination. In which position will the nurse assist the client?

Lithotomy position

The nurse performs a physical examination on a client diagnosed with acute pyelonephritis to assist in determining which of the following?

Location of discomfort The physical examination of a client with pyelonephritis helps the nurse determine the location of discomfort and signs of fluid retention, such as peripheral edema or shortness of breath. Observing and documenting the characteristics of the client's urine helps the nurse detect abnormalities in the urine. Laboratory blood tests reveal elevated calcium levels, whereas radiography and ultrasonography depict structural defects in the kidneys.

Which of the following options are available to women with an increased risk of developing breast cancer?

Long-term follow-up

A nurse who is taking care of a patient with a spinal cord injury documents the frequency of reflex incontinence. The nurse understands that this is most likely due to:

Loss of motor control of the detrusor muscle. Spinal cord injury patients commonly experience reflex incontinence because they lack neurologically mediated motor control of the detrusor and the sensory awareness of the urge to void. These patients also experience hyperreflexia in the absence of normal sensations associated with voiding.

Examination of a client's bladder stones reveals that they are primarily composed of uric acid. The nurse would expect to provide the client with which type of diet?

Low purine A low-purine diet is used for uric acid stones, although the benefits are unknown. Clients with a history of calcium oxalate stone formation need a diet that is adequate in calcium and low in oxalate. Only clients who have type II absorptive hypercalciuria—approximately half of the clients—need to limit calcium intake. Usually, clients are told to increase their fluid intake significantly, consume a moderate protein intake, and limit sodium. Avoiding excessive protein intake is associated with lower urinary oxalate and lower uric acid levels. Reducing sodium intake can lower urinary calcium levels.

A patient who has been treated for uric acid stones is being discharged from the hospital. What type of diet does the nurse discuss with the patient?

Low-purine diet For uric acid stones, the patient is placed on a low-purine diet to reduce the excretion of uric acid in the urine. Foods high in purine (shellfish, anchovies, asparagus, mushrooms, and organ meats) are avoided, and other proteins may be limited.

Sentinel lymph node mapping is done to validate the lack of lymph node metastasis. Which of the following complications does this technique help avoid?

Lymphedema

The nurse is providing care to a client who has had surgery as treatment for breast cancer. The nurse would be alert for the development of which of the following?

Lymphedema

In educating a patient with PMS about changing her dietary practices, what would the nurse recommend that she increase her intake of?

Magnesium

A client undergoes surgery to remove a malignant tumor followed by a urinary diversion procedure. Which postoperative procedure is the most important for the nurse to perform?

Maintain skin and stomal integrity. The most important nursing management in postoperative procedure is to maintain skin and stomal integrity to avoid further complications, such as skin infections and urinary odor.

A nurse is preparing a presentation for a health fair about preventing breast cancer. Which of the following would the nurse include?

Maintaining an ideal weight

A woman comes to the emergency department reporting vaginal discharge and pelvic pain that increases with urination. The client also reports nausea and vomiting, headache, and anorexia. A pelvic examination reveals cervical motion tenderness. The client is diagnosed with acute pelvic inflammatory disease (PID) and is admitted. Which of the following would be most appropriate to include in this client's plan of care?

Maintaining bedrest in the semi-Fowler's position The hospitalized client with pelvic inflammatory disease (PID) is maintained on bed rest and is placed in the semi-Fowler's position to facilitate dependent drainage. Heat may be applied to the abdomen to provide comfort and pain relief. Standard precuations are followed to minimize the risk of infection transmission. PID most commonly results from sexual transmission. An indwelling urinary catheter typically is not necessary unless the client's infection is severe and urinary output must be monitored closely and frequently.

A client is considering breast augmentation. Which of the following would the nurse recommend to the client to ensure that there are no malignancies?

Mammogram

Which of the following dinner selections demonstrates an understanding of nutritional therapy used by women to decrease the signs and symptoms of menopause?

Wheat toast, apple slices, broiled chicken breast, and steamed carrots

The nurse is conducting a history and assessment related to a client's incontinence. Which element should the nurse include in the assessment before beginning a bladder training program?

Medication usage It is essential to assess the client's physical and environmental conditions before beginning a bladder training program, because the patient may not be able to reach the bathroom in time. During the bladder training program, a change in environment may be an effective suggestion for the client. It is not so essential to assess the client's history of allergy, occupation, and smoking habits before beginning a bladder training program.

The nurse is preparing a presentation for an older adult group of males at a senior center. Which of the following would the nurse expect to include when describing the effects of aging on the male reproductive system?

Men retain the ability to fertilize ova irrespective of age.

Which drug is the most effective treatment for trichomoniasis?

Metronidazole The most effective treatment for trichomoniasis is metronidazole. Miconazole, clindamycin, and clotrimazole are not the most effective treatment for trichomoniasis.

A patient asks the nurse if there are any available nonsurgical options to terminate a pregnancy if she is only 2 weeks pregnant. What information should the nurse provide to the patient about a medication that blocks progesterone?

Mifepristone (RU-486, Mifeprex) is used only in early pregnancy to terminate a pregnancy nonsurgically.

The nurse is reviewing the medical record of a client who has come to the clinic for contraception. The nurse determines that hormonal contraceptives would be inappropriate based on the client's history of which of the following?

Migraine headaches with visual auras

A patient has been newly diagnosed with breast cancer. During her preoperative instructions the physician indicated that removal of the breast tissue and axillary lymph node dissection leaving the muscular structure intact was indicated as surgical treatment for her breast cancer. To which of the following procedures is the physician asking the patient to consent?

Modified radical mastectomy

A patient has been newly diagnosed with breast cancer. During her preoperative instructions the physician indicated that removal of the breast tissue and axillary lymph node dissection leaving the muscular structure intact was indicated as surgical treatment for her breast cancer. To which of the following procedures is the physician asking the patient to consent? a) Total mastectomy b) Radical mastectomy c) Segmental mastectomy d) Modified radical mastectomy

Modified radical mastectomy Correct Explanation: A modified radical mastectomy leaves the pectoralis major and minor muscles intact. In a segmental mastectomy, varying amounts of breast tissue are removed, including the malignant tissue and some surrounding tissue to ensure clear margins. In a total mastectomy, breast tissue only is removed. Radical mastectomy includes removal of the pectoralis major and minor muscles in addition to breast tissue and axillary lymph node dissection.

Which of the following terms is used to describe removal of the breast tissue and an axillary lymph node dissection leaving muscular structure intact as surgical treatment of breast cancer? a) Radical mastectomy b) Total mastectomy c) Segmental mastectomy d) Modified radical mastectomy

Modified radical mastectomy Correct Explanation: A modified radical mastectomy leaves the pectoralis major and minor muscles intact. In a segmental mastectomy, varying amounts of breast tissue are removed, including the malignant tissue and some surrounding tissue to ensure clear margins. In a total mastectomy, breast tissue only is removed. Radical mastectomy includes removal of the pectoralis major and minor muscles in addition to breast tissue and axillary lymph node dissection.

A client is being treated for renal calculi and suspected hydronephrosis. Which measure should the nurse take to help maintain a record of the kidneys' function?

Monitor the client's intake and output. Monitoring and recording the client's intake and output provides information about the kidneys' function. It also helps identify any arising complications such as hydronephrosis.

A patient has had surgery to create an ileal conduit for urinary diversion. What is a priority intervention by the nurse in the postoperative phase of care?

Monitor urine output hourly and report output less than 30 mL/hr. In the immediate postoperative period, urine volumes are monitored hourly. Throughout the patient's hospitalization, the nurse monitors closely for complications, reports signs and symptoms of them promptly, and intervenes quickly to prevent their progression. If urinary drainage stops or decreases to less than 30 mL/hour, or if the client complains of back pain, the nurse needs to notify the physician immediately.

The nurse recognizes which of the following statements as accurately reflecting a risk factor for breast cancer?

Mother affected by cancer before 60 years of age

The nurse recognizes which of the following statements as accurately reflecting a risk factor for breast cancer? a) Onset of menses before 14 years of age b) Multiparity c) Mother affected by cancer before 60 years of age d) No alcohol consumption

Mother affected by cancer before 60 years of age Correct Explanation: Risk for breast cancer increases twofold if first-degree female relatives (sister, mother, or daughter) have had breast cancer. Increased risk is associated with early menarche (i.e. menses beginning before 12 years of age). Nulliparity and later maternal age for first birth are associated with increased risk for breast cancer. Alcohol use remains controversial; however, a slightly increased risk is found in women who consume even one drink daily and doubles among women drinking three drinks daily.

Which of the following would be inconsistent as a risk factor for breast cancer?

Multiparity

Which medication may be ordered to relieve discomfort associated with a urinary tract infection?

Phenazopyridine Phenazopyridine is a urinary analgesic ordered to relieve discomfort associated with a UTI. Nitrofurantoin, ciprofloxacin, and levofloxacin are antibiotics.

A client has a history of dysmenorrhea. During monthly menses, the client experiences incapacitating cramping and passes large clots. The client's primary care physician initiates conservative treatment. What interventions would the physician to recommend?

NSAIDs Dysmenorrhea is treated with mild non-narcotic analgesics and by treating the underlying cause if one is identified. Symptomatic relief is accomplished with NSAIDs, which reduce prostaglandins. Prostaglandins are biologic chemicals that exist in endometrial tissue, where they exert a stimulating effect on the uterus, producing cramping and pain.

A nurse has been asked to speak to a local women's group about preventing cystitis. Which of the following would the nurse include in the presentation?

Need to urinate after engaging in sexual intercourse Measures to prevent cystitis include voiding after sexual intercourse, wearing cotton underwear, urinating every 2 to 3 hours while awake, and taking showers instead of tub baths.

An early sign of Paget's disease includes which of the following? a) Increased pulse b) Thickening of areola c) Fever d) Nipple erythema

Nipple erythema Explanation: Early signs of Paget's disease include nipple and areola erythema. Late signs include thickening, scaling, and erosion of the nipple and areola.

Treatment of metabolic acidosis in chronic renal failure includes:

No treatment

A nurse is planning a group teaching session on the topic of urinary tract infection (UTI) prevention. Which point should the nurse include?

Notify the physician if urinary urgency, burning, frequency, or difficulty occurs. Urgency, burning, frequency, and difficulty urinating are all common symptoms of a UTI. The client should notify the physician so that a microscopic urinalysis can be done and appropriate treatment can be initiated. The client should be instructed to drink 2 to 3 L of fluid per day to dilute the urine and reduce irritation of the bladder mucosa. To prevent UTI recurrence, the full amount of antibiotics ordered must be taken despite the fact that the symptoms may have subsided. Women are told to avoid scented toilet tissue and bubble baths and to wear cotton underwear, not nylon, to reduce the chance of irritation.

The nurse is reviewing the medical record of a client diagnosed with uterine cancer. Which of the following, if noted, would the nurse identify as a risk factor?

Nulliparity Risk factors for uterine cancer include age of 55 years or more, obesity, unopposed estrogen therapy, nulliparity, truncal obesity, late menopause (after age 52 years), and use of tamoxifen. Early age at first sexual intercourse, exposure to diethylstilbesterol, and vitamin deficiencies are risk factors associated with cervical cancer.

The nurse cares for a client with a right-arm arteriovenous fistula (AVF) for hemodialysis treatments. Which nursing action is contraindicated?

Obtaining a blood pressure reading from the right arm

The nurse teaches the female patient who is premenopausal to perform breast self-examination (BSE) at which time frame?

On day 5 to day 7, counting the first day of menses as day 1

A client with breast cancer is receiving chemotherapy as part of the treatment plan. The client develops neutropenia and is scheduled to receive pegfilgrastim (Neulasta). The nurse would expect this drug to be given at which time? a) Once, approximately 24 hours after chemotherapy b) Weekly during chemotherapy administration c) Every 2 to 3 weeks after chemotherapy is given d) 7 to 10 days after chemotherapy administration

Once, approximately 24 hours after chemotherapy Explanation: Pegfilgrastim (Neulasta) is a long-acting granulocyte colony-stimulating factor(G-CSF) that is given via injection once, 24 hours after chemotherapy. Filgrastim (Neupogen), a short-acting G-CSF, is injected subcutaneously from 7 to 10 days after chemotherapy administration. Epoetin alfa, an erythropoietin growth factor, is administered weekly; darbepoetin alfa, a long-acting erythorpoietin growth factor, is administered every 2 to 3 weeks.

The nurse is completing the admission history for a client who is admitted for a reduction mammoplasty. Which of the following client statements is uncommon when explaining the rationale for the procedure?

Others disapprove

Which of the following is the most common symptom of bladder cancer?

Painless gross hematuria Painless gross hematuria is the most common symptom of bladder cancer. Pelvic and back pain may occur with metastasis. Any alteration in voiding or change in the urine may indicate cancer of the bladder.

A nurse is reviewing the history and physical examination of a client with a suspected malignant tumor of the bladder. Which finding would the nurse identify as the most common initial symptom?

Painless hematuria The most common first symptom of a malignant tumor of the bladder is painless hematuria. Additional early symptoms include UTI with symptoms such as fever, dysuria, urgency, and frequency. Later symptoms are related to metastases and include pelvic pain, urinary retention (if the tumor blocks the bladder outlet), and urinary frequency from the tumor occupying bladder space.

A 64-year-old man is seeing his urologist for an annual check-up, post prostatectomy. The health care provider is concerned with the symptom he finds because it is considered diagnostic for bladder cancer. Which of the following signs/symptoms is diagnostic for bladder cancer?

Painless, gross hematuria Although flank pain may occur, the painless, gross hematuria is characteristic of bladder cancer.

A 32-year-old woman has had three consecutive normal Pap tests. Which of the following recommendations for screening would be appropriate?

Pap testing every 2 to 3 years

A patient reports a pain rating of 6 (on a 1 to 10 pain scale, with 10 being the worst possible pain) when the nurse examiner gently moves the cervix. This finding is most consistent with which of the following conditions?

Pelvic infection

A nurse is providing care to a client who was just admitted with a diagnosis of ectopic pregnancy. When reviewing the client's history, which of the following would the nurse identify as a major risk factor for this client?

Pelvic inflammatory disease (PID)

When assessing the impact of medications on the etiology of acute renal failure, the nurse recognizes which of the following as the drug that is not nephrotoxic?

Penicillin

If an indwelling catheter is necessary, which nursing intervention should be implemented to prevent infection?

Perform meticulous perineal care daily with soap and water Cleanliness of the area will reduce potential for infection. Strict aseptic technique must be used when inserting a urinary bladder catheter. The nurse must maintain a closed system and use the catheter's port to obtain specimens. The catheter bag must never be placed on the client's abdomen unless it is clamped because it may cause urine to flow back from the tubing into the bladder.

A female client is undergoing a bladder training program as treatment for urinary incontinence. Which of the following techniques would be the most appropriate suggestion?

Performing Kegel exercises. Instructing the client on Kegel exercises will help her achieve continence. These exercises improve muscle tone and voluntary control. Bladder instillation of DMSO and referring the client to a chronic pain center are therapies to manage interstitial cystitis. Warm sitz baths may be suggested to a client in the event of urethra inflammation.

The nurse is preparing a teaching plan for a client with a vulvovaginal infection. Which of the following would be least appropriate for the nurse to include?

Performing douching with a dilute vinegar solution twice a day Research has shown that douching provides no benefit in the prevention or care of vulvovaginal infections. Douching usually is unnecessary because daily baths or showers and proper hygiene after voiding and defecation keep the perineal area clean. In addition, douching tends to eliminate normal flora, reducing the body's ability to ward off infection. Repeated douching may result in vaginal epithelial breakdown and chemical irritation. The client should recline for approximately 30 minutes after inserting any vaginal medication to prevent the medication from escaping from the vagina. Loose-fitting cotton underwear is advised rather than tight-fitting synthetic, nonabsorbent, heat-retaining underwear. Unprotected sexual intercourse is associated with risks and should be avoided.

While caring for a client who is being treated for severe pelvic inflammatory disease (PID), which nursing action minimizes transmission of infection?

Performing hand hygiene when entering the room While caring for a client hospitalized with PID, the nurse has to reduce the risk of the spread of pathogenic microorganisms. Meticulous hand hygiene minimizes the transmission of infection. There is no reason to restrict visitors in this situation; however, visitors need to be educated in proper hand hygiene. Reverse isolation is not indicated for this client. A sitting position will not aid in preventing nosocomial infections.

Which of the following is the period of transition from normal periods to the complete cessation of menses for 1 year?

Perimenopause

The nurse is caring for a postoperative client who has a Kock pouch. Nursing assessment findings reveal abdominal pain, absence of bowel sounds, fever, tachycardia, and tachypnea. The nurse suspects which of the following?

Peritonitis Clinical manifestations of peritonitis include abdominal pain and distention, absence of bowel sounds, nausea and vomiting, fever, changes in vital signs.

An 82-year-old client experiences urinary incontinence. Which factor should the nurse assess before beginning a bladder training program for this client?

Physical and environmental conditions It is essential to assess the client's physical and environmental conditions before beginning a bladder training program, because the client may not be able to reach the bathroom in time. During the bladder training program, a change in environment may be an effective suggestion for the client. It is not so essential to assess the client's history of allergy, occupation, and smoking habits before beginning a bladder training program.

Which characteristic is seen with a healthy stoma?

Pink color Characteristics of a normal stoma include a pink and moist appearance. It is insensitive to pain because it has no nerve endings. The area is vascular and may bleed when cleaned.

As a grade school nurse, you speak to the sixth grade boys regarding their physical maturation and body changes during puberty. In your student education sessions, you discuss the procedure of circumcision and how it is viewed by different ethnicities. Which external reproductive structure is removed by circumcision?

Prepuce

Following a radical vulvectomy, the nurse is preparing the client for discharge to home. Which care intervention would be considered the priority for this client?

Prevention of wound complications Prevention of complications and infection is paramount due to the location of the wound. The perineal area provides a warm, dark environment that supports the growth of microorganisms that can be introduced into the wound. With a radical vulvectomy, it is likely to find surgical drains and urinary catheter that can also interfere with the maintenance of the wound. Relieving edema to the lower extremities may be a necessary part of care if the lymph nodes and blood vessels are disturbed. The client may have a colostomy and care instructions should be provided. Alteration in sexual function needs to be addressed but not a priority for the initial stage of healing.

During menstruation there is an absence of which hormone?

Progesterone

Ovulation occurs midway between days 17 and 22 (usually at day 14) in a normal 28-day menstrual cycle. Conception occurs during ovulation. If an egg is fertilized, this hormone will continue to be produced to protect the egg until the placenta takes over. Which of the following is the correct hormone?

Progesterone

In the menstrual cycle, every month the female reproductive system generates an ovum. When the ovum is not fertilized, production of which of the following leads to menstruation?

Progesterone by the corpus luteum beginning to decrease

A patient is diagnosed with hypertension. In addition, the patient reports skin discoloration, weight gain, and nausea. Which of the following contraceptive preparations would the nurse practitioner recommend for this patient?

Progestin-only

A nurse is employed by a fertility clinic. Diagnostic testing is being completed to determine the source of the male not producing viable sperm to enable ovum fertilization. Which male structures does the nurse consider if the problem is narrowed to sperm survivability? You Selected:

Prostate gland

A client has been diagnosed with acute glomerulonephritis. This condition causes:

Proteinuria

What is a hallmark of the diagnosis of nephrotic syndrome?

Proteinuria

A patient is considering use of chemoprevention because she is at high risk for developing breast cancer. What can the nurse do to assist the patient with her decision?

Provide the patient with information regarding the benefits, risks, and possible side effects.

Which term refers to inflammation of the renal pelvis?

Pyelonephritis Pyelonephritis is an upper urinary tract inflammation, which may be acute or chronic. Cystitis is inflammation of the urinary bladder. Urethritis is inflammation of the urethra. Interstitial nephritis is inflammation of the kidney.

A patient with a UTI is having burning and pain when urinating. What urinary analgesic is prescribed for relief of these symptoms?

Pyridium The urinary analgesic agent phenazopyridine (Pyridium) is used specifically for relief of burning, pain, and other symptoms associated with UTI.

Which laboratory value supports a diagnosis of pyelonephritis?

Pyuria Pyelonephritis is diagnosed by the presence of pyuria, leukocytosis, hematuria, and bacteriuria. The client exhibits fever, chills, and flank pain. Myoglobinuria is seen with any disease process that destroys muscle. Ketonuria indicates a diabetic state. Because the client with pyelonephritis typically has signs of infection, the WBC count is more likely to be high rather than low.

A nurse is reviewing the history of a client who is suspected of having glomerulonephritis. Which of the following would the nurse consider significant?

Recent history of streptococcal infection

When the nurse places the patient in the stirrups for a pelvic exam she observes a bulge caused by rectal cavity protrusion. What does the nurse know this protrusion is called?

Rectocele

On a follow-up visit, the patient is informed that her breast cancer has recurred. The nurse knows which of the following situations places the patient at risk for developing bone metastases?

Recurrence within 2 years of the original diagnosis

A nurse is caring for a client who has just had a modified radical mastectomy with immediate reconstruction. She's in her 30s and has two young children. Although she's worried about her future, she seems to be adjusting well to her diagnosis. What should the nurse do to support her coping?

Refer the client to the American Cancer Society's Reach for Recovery program or another support program.

Sympathomimetics have which of the following effects on the body?

Relaxation of bladder wall Sympathomimetics mimic the sympathetic nervous system, causing increased heart rate and contractility, dilation of bronchioles and pupils, and bladder wall relaxation.

A woman comes to her health care provider's office with signs and symptoms of kidney stones. Which of the following should be the primary medical management goal?

Relieve the pain. The immediate objective is to relieve pain, which can be incapacitating depending on the location of the stone.

A patient has been diagnosed with postrenal failure. The nurse reviews the patient's electronic health record and notes a possible cause. Which of the following is the possible cause?

Renal calculi

One of the roles of the nurse in caring for clients with chronic renal failure is to help them learn to minimize and manage potential complications. This would include:

Restricting sources of potassium usually found in fresh fruits and vegetables

A client with renal failure is undergoing continuous ambulatory peritoneal dialysis. Which nursing diagnosis is the most appropriate for this client?

Risk for infection

Following percutaneous nephrolithotomy, the client is at greatest risk for which nursing diagnosis?

Risk for infection Percutaneous nephrolithotomy is an invasive procedure for the removal of renal calculi. The client would be at risk for infection.

As a grade school nurse, you speak to the sixth grade boys regarding their physical maturation and body changes during puberty. In your student education sessions, you discuss male anatomy and the appropriate titles and functions of each structure. Which structure is involved in keeping the testes at the necessary temperature to ensure sperm production?

Scrotum

A perimenopausal woman informs the nurse that she is having irregular vaginal bleeding. What should the nurse encourage the patient to do?

See her gynecologist as soon as possible. All women should be encouraged to have annual checkups, including a gynecologic examination. Any woman who is experiencing irregular bleeding should be evaluated promptly.

Which of the following would a nurse classify as a prerenal cause of acute renal failure?

Septic shock

A 35-year-old female client is requesting information about mammograms and breast cancer. She isn't considered at high risk for breast cancer. What should the nurse tell this client?

She should eat a low-fat diet to further decrease her risk of breast cancer.

Which of the following positions may be utilized for a patient who is unable to maintain the supine lithotomy position?

Sims'

During a physical examination, the nurse inspects the external genitalia of a male client. Which of the following would the nurse identify as an abnormality?

Skin lesions

What is used to decrease potassium level seen in acute renal failure?

Sodium polystyrene sulfonate

A client diagnosed with acute kidney injury (AKI) has a serum potassium level of 6.5 mEq/L. The nurse anticipates administering:

Sodium polystyrene sulfonate (Kayexalate)

Following a nephrectomy, which assessment finding is most important in determining nursing care for the client?

SpO2 at 90% with fine crackles in the lung bases

Nursing students are reviewing information about natural family planning methods in preparation for a quiz the next day. The students demonstrate a need for additional study when they identify which of the following as an example?

Spermicide

A client is admitted with nausea, vomiting, and diarrhea. His BP is 74/30 mmHg. The client is oliguric and his blood urea nitrogen (BUN) and creatinine levels are elevated. The physician will most likely write an order for which treatment?

Start IV fluids with a normal saline solution bolus followed by a maintenance dose

Which of the following is one of the most important prognostic factor in breast cancer? a) Age of patient b) Obesity c) Family history d) Status of lymph nodes

Status of lymph nodes Explanation: The two most important prognostic factor in breast cancer is the status of the lymph nodes and tumor size. Obesity, age of the patient, and family history are not the most important prognostic factors in breast cancer.

Which type of biopsy is used for nonpalpable lesions found on mammography? a) Tru-Cut core b) Incisional c) Stereotactic d) Excisional

Stereotactic Correct Explanation: Stereotactic biopsy utilizes computer location of the suspicious area found on biopsy, followed by core needle insertion and sampling of tissue for pathologic examination. An excisional biopsy is the usual procedure for any palpable breast mass. Incisional biopsy is performed on a palpable mass when tissue sampling alone is required. Tru-Cut core biopsy is used when a tumor is relatively large and close to the skin surface.

Which of the following nursing actions is most important in caring for the client following lithotripsy?

Strain the urine carefully for stone fragments. The nurse should strain all urine following lithotripsy. Stone fragments are sent to the laboratory for chemical anaysis.

Which type of incontinence refers to involuntary loss of urine through an intact urethra as a result of a sudden increase in intra-abdominal pressure?

Stress Stress incontinence may occur with sneezing, coughing, or changing position. Overflow incontinence refers to the involuntary loss of urine associated with overdistention of the bladder. Urge incontinence refers to involuntary loss of urine associated with urgency. Reflex incontinence refers to the involuntary loss of urine due to involuntary urethral relaxation in the absence of normal sensations.

A patient informs the nurse that every time she sneezes or coughs, she urinates in her pants. What type of incontinence does the nurse recognize the patient is experiencing?

Stress incontinence Stress incontinence is the involuntary loss of urine through an intact urethra as a result of sneezing, coughing, or changing position (Meiner, 2011; Miller, 2012).

The nurse is educating a female patient with a UTI on the pharmacologic regimen for treatment. What is important for the nurse to instruct the patient to do?

Take the antibiotic for 3 days as prescribed. The trend is toward a shortened course of antibiotic therapy for uncomplicated UTIs, because most cases are cured after 3 days of treatment. Regardless of the regimen prescribed, the patient is instructed to take all doses prescribed, even if relief of symptoms occurs promptly. Although brief pharmacologic treatment of UTIs for 3 days is usually adequate in women, infection recurs in about 20% of women treated for uncomplicated UTIs.

Hyperkalemia is a serious side effect of acute renal failure. Identify the electrocardiogram (ECG) tracing that is diagnostic for hyperkalemia.

Tall, peaked T waves

A health care provider tells a breast cancer patient that he is going to prescribe hormone therapy that has been found to significantly reduce mortality. Which of the following is the drug that would most likely be prescribed?

Tamoxifen

A nurse who works in an oncology practice prepares patients for the side effects of adjuvant hormonal therapy to treat breast cancer. Which of the following is the hormonal agent that has an increased risk of pulmonary embolism and deep vein thrombosis?

Tamoxifen

Students are reviewing information about the use of adjuvant hormonal therapy for breast cancer. They demonstrate understanding of this information when they identify which of the following as an example of a selective estrogen receptor modulator (SERM)?

Tamoxifen

Which nursing intervention should the nurse caring for the client with pyelonephritis implement?

Teach client to increase fluid intake up to 3 liters per day. The nurse teaches the client to increase fluid intake to promote renal blood flow and flush bacteria from the urinary tract.

A client has undergone a renal transplant and returns to the health care agency for a follow-up evaluation. Which finding would lead to the suspicion that the client is experiencing rejection?

Tenderness over transplant site

A client has been admitted for an outpatient cystoscopy because of a suspected interstitial cystitis. Which statement best describes the pathology of this disorder?

The bladder wall contains multiple pinpoint hemorrhagic areas that join and form larger hemorrhagic areas that may progress to fissuring and scarring of the bladder mucosa. With interstitial cystitis, the bladder wall contains multiple pinpoint hemorrhagic areas that join and form larger hemorrhagic areas that may progress to fissuring and scarring of the bladder mucosa. A common cause of urethritis in men is infection with Chlamydia trachomatis. Cystitis is usually caused by bacterial infection. The surface of the bladder becomes edematous and reddened, and ulcerations may develop. With urinary incontinence, the bladder can contract without warning, fail to accommodate adequate volumes of urine, or fail to empty completely.

When a client with an indwelling urinary catheter wants to walk to the hospital lobby to visit with family members, the nurse teaches him how to do this without compromising the catheter. Which client action indicates an accurate understanding of this information?

The client keeps the drainage bag below the bladder at all times. To maintain effective drainage, the client should keep the drainage bag below the bladder; doing so allows the urine to flow by gravity from the bladder to the drainage bag. The client shouldn't lay the drainage bag on the floor because the bag could become grossly contaminated. The client shouldn't clamp the catheter drainage tubing because this impedes the flow of urine. To promote drainage, the client may loop the drainage tubing above — not below — its point of entry into the drainage bag.

Which of the following is an expected outcome for a client 24 hours after an abdominal hysterectomy?

The client will perform leg exercises hourly. During the first 24 hours after an abdominal hysterectomy, the client is at risk for development of thrombophlebitis because of potential interference with pelvic and leg circulation. Leg exercises are essential to promote circulation and prevent a thrombus. Bowel sounds may not be heard immediately after surgery. It may take up to 48 hours for peristalsis to return. Perineal pads are used after a vaginal hysterectomy, not an abdominal hysterectomy. In the early phases of recovery, the client will be more likely to focus on expressing feelings of discomfort rather than a positive body image.

A client comes to the clinic for a routine evaluation. During the physical examination, the nurse palpates the client's breast and finds a small lump. Which of the following would lead the nurse to suspect possible breast cancer?

The lump is irregularly shaped.

The client arrives at a public health clinic worried that she has breast cancer since finding a lump in her breast. When assessing the breast, which assessment finding is characteristic of fibrocystic disease?

The lump is round and movable.

A patient is having a fine-needle biopsy (FNB) for a mass in the left breast. When the needle is inserted and the mass is no longer palpable, what does the nurse know has most likely occurred?

The mass may be cystic and was ruptured when the needle was inserted.

The nurse is outlining the female internal reproductive structures on a diagram. Where on the diagram would the nurse highlight the typical site of ovum fertilization?

The nurse would highlight the fallopian tube.

The nurse is assisting in the transport of a client with an indwelling catheter to the diagnostic studies unit. Which action made by the nursing assistant would require instruction?

The nursing assistant places the drainage bag on the client's abdomen for transport. The nurse would instruct the nursing assistant to maintain the drainage bag lower than the genital region to avoid a backflow of urine into the bladder. The nursing assistant is correct to move the catheter and drainage bag with the client to not put tension on the catheter, place the drainage bag on the lower area of the wheelchair, and hold the drainage bag while the client is in the process of moving.

A patient has had a total mastectomy 12 hours ago and the nurse is assessing the surgical wound. The nurse observes ecchymosis, swelling, and tightness around the wound, and the patient states that it is painful. What does the nurse suspect has occurred?

The patient has developed a hematoma.

A client with bladder cancer had his bladder removed and an ileal conduit created for urine diversion. While changing this client's pouch, the nurse observes that the area around the stoma is red, weeping, and painful. What should the nurse conclude?

The pouch faceplate doesn't fit the stoma If the pouch faceplate doesn't fit the stoma properly, the skin around the stoma will be exposed to continuous urine flow from the stoma, causing excoriation and red, weeping, and painful skin. A lubricant shouldn't be used because it would prevent the pouch from adhering to the skin. When properly applied, a skin barrier prevents skin excoriation. Stoma dilation isn't performed with an ileal conduit, although it may be done with a colostomy if ordered.

When instructing clients at a prenatal class, the nurse is most correct to state which medical name as being removed during circumcision?

The prepuce

The nurse is caring for a client who is ordered a sentinel lymph node biopsy. The physician explained the procedure and desired outcome. Which statement, made by the client, indicates a need for further instruction?

The procedure removes all cancer from the body.

A 21-year-old female is talking with a nurse at her gynecologist's office. She is scheduled to have her first Pap test and she asks the nurse why she needs to have this test done. Which of the following is the nurse's best response?

The test is used to detect early cancer of the cervix and to determine estrogen activity as it relates to menopause or endocrine abnormalities.

The nurse is caring for a client with recurrent urinary tract infections. Which of the following body structures would the nurse instruct as the most frequent cause of women's urinary tract infections?

The urethra Because the urethra is short in women, ascending infections or microorganisms carried from the vagina or rectum are common. Males have a longer urethra, causing the organisms travel farther to the bladder. Although structures of the urinary system, the other options are where the client has bacteria and microorganisms located. The ureters connect the bladder to kidney thus do not obtain bacteria, just transmit when available.

A client with breast cancer is scheduled to undergo chemotherapy with aromatase inhibitors. Which of the following best reflects the rationale for using this group of drugs?

They lower the level of estrogen in the body blocking the tumor's ability to use it.

A client with breast cancer is scheduled to undergo chemotherapy with aromatase inhibitors. Which of the following best reflects the rationale for using this group of drugs? a) They lower the level of estrogen in the body blocking the tumor's ability to use it. b) They stimulate the immune system to attack a protein common in many tumors. c) They block progesterone-dependent tumors from growing. d) They attach to endogenous protein receptors to slow the growth of cancerous cells.

They lower the level of estrogen in the body blocking the tumor's ability to use it. Explanation: Aromatase inhibitors lower the level of estrogen in the body thereby interfering with the ability of hormone-sensitive tumors to use estrogen for growth. Antiprogestin drug, such as mifepristone, blocks progesterone-dependent breast cancers. The monoclonal antibody, trastuzumab attaches to protein receptors to slow the growth of cancer cells. A breast cancer vaccine is under investigation in Italy. This vaccine stimulates the immune system to attack a protein called mammaglobin-A, which is found in 80% of breast cancer tumors.

A female client has just been diagnosed with condylomata acuminata (genital warts). What information is appropriate to tell this client?

This condition puts her at a higher risk for cervical cancer; therefore, she should have a Papanicolaou (Pap) smear annually. Women with condylomata acuminata are at risk for cancer of the cervix and vulva. Yearly Pap smears are very important for early detection. Because condylomata acuminata is a virus, there is no permanent cure. Because condylomata acuminata can occur on the vulva, a condom won't protect sexual partners. HPV can be transmitted to other parts of the body, such as the mouth, oropharynx, and larynx.

A client complains of having tender and painful breasts, often feeling multiple lumps within her breast tissue. The nurse would need to gather additional information about which of the following?

Timing of symptoms in relation to the menstrual cycle

A urologist suspects that a 62-year-old male client might have prostate cancer. Which of the following tests would be used to make a diagnosis of cancer?

Tissue biopsy

The nursing is caring for a client who will be having artificial implants for breast reconstruction. The client is arriving at the physician's office for which procedure completed before the surgery can be done?

Tissue expansion

The nursing is caring for a client who will be having artificial implants for breast reconstruction. The client is arriving at the physician's office for which procedure completed before the surgery can be done?

Tissue expansion Correct Explanation: Before an implant for breast reconstruction can produce an optimum cosmetic appearance, the skin and tissue on the chest wall are expanded to provide a large enough space to fill and approximate the size of the remaining breast. The other options are not correct.

A patient diagnosed with endometriosis asks for an explanation of the disease. What should the nurse explain to the patient?

Tissue from the lining of the uterus has implanted in areas outside the uterus. Endometriosis is a chronic disease affecting between 6% and 10% of women of reproductive age (Falcone & Lebovic, 2011) and consisting of a benign lesion or lesions that contain endometrial tissue (similar to that lining the uterus) found in the pelvic cavity outside the uterus.

Which therapy uses low-level radiofrequencies to produce localized heat that destroys prostate tissue?

Transurethral needle ablation

During an internal vaginal examination, the nurse practitioner notes a frothy and malodorous discharge. The nurse suspects the odor is caused by which bacteria?

Trichomonas

When a female client reports a frothy, yellow-green vaginal discharge, the nurse suspects the client has a vaginal infection caused by which organism?

Trichomonas vaginalis Trichomonas vaginalis causes a frothy yellow-white or yellow-green vaginal discharge. Candidiasis causes a white, cheeselike discharge clinging to the vaginal epithelium. Gardnerella vaginalis causes a gray-white to yellow-white discharge clinging to the external vulva and vaginal walls. Chlamydia causes a profuse purulent discharge.

Chance Henderson, a nine-month-old male, is being seen in the urology office where you practice nursing. Chance demonstrates cryptorchidism and the surgeon and his parents are developing a treatment strategy. By what age should surgery be performed to avoid permanent consequences?

Two years

Breast cancers are usually found in which area of the breast?

Upper outer quadrant

What is the most common location for breast cancer?

Upper outer quadrant of the breast

During a teaching demonstration on BSE, the nurse should always explain that the majority of breast cancers are found in the:

Upper, outer quadrant.

A client comes to the clinic for a follow-up visit. During the interview, the client states, "Sometimes when I have to urinate, I can't control it and do not reach the bathroom in time." The nurse suspects that the client is experiencing which type of incontinence?

Urge Urge incontinence occurs when the client experiences the sensation to void but cannot control voiding in time to reach a toilet. Stress incontinence occurs when the client has an involuntary loss of urine that results from a sudden increase in intra-abdominal pressure. Overflow incontinence occurs when the client experiences an involuntary loss of urine related to an over distended bladder; the client voids small amounts frequently and dribbles. Functional incontinence occurs when the client has function of the lower urinary tract but cannot identify the need to void or ambulate to the toilet.

The nurse advises a patient with renal stones to avoid eating shellfish, asparagus, and organ meats. She emphasizes these foods because she knows that his renal stones are composed of which of the following substances?

Uric acid Uric acid stones are found in patients with gout and myeloproliferative disorders. Therefore, a diet low in purines is recommended.

A client comes to the emergency department complaining of a sudden onset of sharp, severe flank pain. During the physical examination, the client indicates that the pain, which comes in waves, travels to the suprapubic region. He states, "I can even feel the pain at the tip of my penis." Which of the following would the nurse suspect?

Urinary calculi Symptoms of a kidney or ureteral stone vary with size, location, and cause. Small stones may pass unnoticed; however, sudden, sharp, severe flank pain that travels to the suprapubic region and external genitalia is the classic symptom of urinary calculi. The pain is accompanied by renal or ureteral colic, painful spasms that attempt to move the stone. The pain comes in waves that radiate to the inguinal ring, the inner aspect of the thigh, and to the testicle or tip of the penis in men, or the urinary meatus or labia in women. Clients with acute glomerulonephritis may be asymptomatic or may exhibit fever, nausea, malaise, headache, edema (generalized or periorbital), pain, and mild to moderate hypertension. Clients with ureteral stricture may complain of flank pain and tenderness at the costovertebral angle and back or abdominal discomfort. A client with renal cell carcinoma rarely exhibits symptoms early on but may present with painless hematuria and persistent back pain in later stages.

Which statement describing urinary incontinence in an older adult client is true?

Urinary incontinence isn't a disease. Urinary incontinence isn't a normal part of aging nor is it a disease. It may be caused by confusion, dehydration, fecal impaction, restricted mobility, or other causes. Certain medications, including diuretics, hypnotics, sedatives, anticholinergics, and antihypertensives, may trigger urinary incontinence. Most clients with urinary incontinence can be treated; some can be cured.

A group of students are reviewing information about disorders of the bladder and urethra. The students demonstrate understanding of the material when they identify which of the following as a voiding dysfunction?

Urinary retention Urinary retention and urinary incontinence are voiding dysfunctions, temporary or permanent alterations in the ability to urinate normally. Cystitis is an infectious disorder. Bladder stones and urethral stricture are obstructive disorders.

Which of the following is the most common site of a nosocomial infection?

Urinary tract The urinary tract is the most common site of nosocomial infection, accounting for greater than 3% of the total number reported by hospitals each year.

A nurse assesses a client shortly after living donor kidney transplant surgery. Which postoperative finding must the nurse report to the physician immediately?

Urine output of 20 ml/hour

A client admitted with a gunshot wound to the abdomen is transferred to the intensive care unit after an exploratory laparotomy. IV fluid is being infused at 150 mL/hour. Which assessment finding suggests that the client is experiencing acute renal failure (ARF)?

Urine output of 250 ml/24 hours

The nurse is reviewing the client's lab results. Which lab result requires follow up by the nurse? Select all that apply.

Urine: RBC 20 BUN 28 mg/dL Hematuria (> 3RBCs) and an elevated BUN are both suggestive of a problem within the genitourinary tract. A serum creatinine of 0.8 mg/dL and a urine specific gravity of 1.020 are within normal limits. A rare white blood cell is not clinically significant.

While examining the introitus, the nurse practitioner asks the patient to "bear down." The nursing student observing the examination knows that the nurse practitioner is assessing the patient for which of the following conditions?

Uterine prolapse

A group of students are reviewing the female reproductive system in preparation for a test. Which of the following if identified by the students as an internal structure indicates successful learning?

Vagina

A client with HIV has recently completed a 7-day regimen of antibiotics. She reports vaginal itching and irritation. In addition, the client has a white, cheeselike vaginal discharge. Which condition is the client most likely experiencing?

Vulvovaginal candidiasis Use of antibiotics decreases bacteria, thereby altering the natural protective organisms usually present in the vagina, which can lead to candidiasis overgrowth. Clinical manifestations include a vaginal discharge that causes pruritus; the discharge may be watery or thick but usually has a white, cheeselike appearance. Bacterial vaginosis does not produce local discomfort or pain. Discharge, if noticed, is heavier than normal and is gray to yellowish white. Most HPV infections are self-limiting and without symptoms.

A male client has doubts about performing peritoneal dialysis at home. He informs the nurse about his existing upper respiratory infection. Which of the following suggestions can the nurse offer to the client while performing an at-home peritoneal dialysis?

Wear a mask when performing exchanges

During a follow-up visit, a female client who underwent a mastectomy asks the nurse if she can work in her backyard or at least do some household work. Which suggestion would be most appropriate?

Wear gloves and protective clothing to avoid any injuries.

Which of the following is the most accurate indicator of fluid loss or gain?

Weight

A patient taking an alpha-adrenergic medication for the treatment of hypertension is having a problem with incontinence. What does the nurse tell the patient?

When the medication is discontinued or changed, the incontinence will resolve. Iatrogenic incontinence refers to the involuntary loss of urine due to extrinsic medical factors, predominantly medications. One such example is the use of alpha-adrenergic agents to decrease blood pressure. In some people with an intact urinary system, these agents adversely affect the alpha receptors responsible for bladder neck closing pressure; the bladder neck relaxes to the point of incontinence with a minimal increase in intra-abdominal pressure, thus mimicking stress incontinence. As soon as the medication is discontinued, the apparent incontinence resolves.

The nurse is assigned to care for a patient in the oliguric phase of kidney failure. When does the nurse understand that oliguria is said to be present?

When the urine output is less than 30 mL/h Oliguria is defined as urine output <0.5 mL/kg/h

Because of difficulties with hemodialysis, peritoneal dialysis is initiated to treat a client's uremia. Which finding during this procedure signals a significant problem?

White blood cell (WBC) count of 20,000/mm^3

A patient is receiving chemotherapy with paclitaxel as treatment for ovarian cancer. The patient arrives at the facility for laboratory testing prior to her next dose of chemotherapy. The results are as follows: Hemoglobin: 12.9 gm/dL White blood cell count: 2,200 /cu mm Platelets: 250,000 /cu mm Red blood cell count: 4,400,00/cu mm Which result would be a cause for concern?

White blood cell count The patient's white blood cell count is low, revealing leukopenia and placing the patient at an increased risk for infection. The other results are within normal parameters and would not be a cause for concern.

The nurse is discussing contraception with a patient interested in transdermal contraceptives. What should the nurse inform the client is the most common side effect of transdermal contraceptives?

Withdrawal bleeding

Which group represents the fastest-growing segment of the population with AIDS?

Women Women represent the fastest-growing segment of the population with AIDS.

A patient has been diagnosed with a Trichomoniasis vaginal infection. The nurse would expect which color of discharge?

Yellow-green

A client who had intracavity radiation treatment for cervical cancer 1 month earlier reports small amounts of vaginal bleeding. This finding most likely represents:

an expected effect of the radiation therapy. After intracavity radiation, some vaginal bleeding occurs for 1 to 3 months. Intermittent, painless vaginal bleeding is a classic symptom of cervical cancer, but given the client's history, bleeding in more likely a result of the radiation. The passage of feces through the vagina, not vaginal bleeding, is a sign of rectovaginal fistula. Vaginal infections are indicated by various types of vaginal discharge, not vaginal bleeding.

Which is the earliest and the most common symptom of endometrial cancer?

bleeding Bleeding is the earliest and the most common sign of endometrial cancer.

A client who has a history of neurogenic bladder uses a permanent, indwelling catheter to facilitate urine elimination. What can this client consume to decrease the likelihood of bladder infection?

cranberry juice Cranberry juice or vitamin C may be recommended to keep the bacteria from adhering to the wall of the bladder and thus promoting their excretion and enhancing the effectiveness of drug therapy.

A client is undergoing a diagnostic workup for suspected testicular cancer. When obtaining the client's history, the nurse checks for known risk factors for this type of cancer. Testicular cancer has been linked to:

cryptorchidism.

A client is scheduled to undergo surgical creation of an ileal conduit. The primary nurse educates the client about surgery and the postoperative period. The nurse informs the client that many members of the health care team (including a mental health practitioner) will see him. A mental health practitioner should be involved in the client's care to:

help the client cope with the anxiety associated with changes in body image. Many clients who undergo surgery for creation of an ileal conduit experience anxiety associated with changes in body image. The mental health practitioner can help the client cope with these feelings of anxiety. Mental health practitioners don't evaluate whether the client is a surgical candidate. None of the evidence suggests that urinary diversion surgery, such as creation of an ileal conduit, places the client at risk for suicide. Although evaluating the need for mental health intervention is always important, this client displays no behavioral changes that suggest intervention is necessary at this time.

The nurse is teaching the client who will undergo surgery for the creation of a nephrostomy. Which of the images best depicts this type of cutaneous urinary diversion?

https://s3.amazonaws.com/prepu/prod/images/4020.jpeg A cutaneous diversion involves the creation of an opening through the abdominal wall and skin to allow urine to drain. A nephrosostomy (Option D) allows urine to drain directly from the kidney through a percutaneous catheter through an opening in the flank. An ileal conduit (Option A) is the most common cutaneous diversion, whereby both ureters empty into an isolated section of the ileum. One end of the isolated segment is brought through the abdominal wall and allows urine to drain through a stoma. With a cutaneous ureterostomy (Option B), the ureter is detached from the bladder and brought through the abdominal wall and attached to an opening in the skin. The bladder is sutured to the abdominal wall and a stoma is created through the abdominal and bladder walls for drainage of urine in a vesicostomy (Option C).

The nurse monitors the client for potential complications during dialysis but recongizes NOT to monitor for

hypertension

Bladder retraining following removal of an indwelling catheter begins with

instructing the client to follow a 2- to 3-hour timed voiding schedule. Immediately after the removal of the indwelling catheter, the client is placed on a timed voiding schedule, usually 2 to 3 hours, not 6 hours. At the given time interval, the client is instructed to void. Immediate voiding is not usually encouraged. If bladder ultrasound shows 100 mL or more of urine remaining in the bladder after voiding, straight catheterization may be performed to ensure complete bladder emptying.

A client who has been unable to conceive is intent on discovering the reason. The client has had very scanty, infrequent menses since their onset, and has a history of type 2 diabetes and acne. What could be the reason for this client's menstrual disorder?

polycystic ovarian syndrome Polycystic ovarian syndrome, a condition characterized by a cluster of signs and symptoms that include amenorrhea and oligomenorrhea, affects women between 20 and 40 years of age. Polycystic ovarian syndrome is associated with multiple endocrine abnormalities such as overproduction and inefficient use of insulin and high testosterone levels. Women with this disorder tend to have problems including weight gain, excessive growth of body hair, acne, thinning hair or baldness, abnormal lipid levels, and hypertension.

A client has developed urinary incontinence and is beginning bladder training to regain control over urine elimination. The catheter would be clamped and unclamped to:

promote normal bladder function. The clamping and unclamping of the catheter begins to reestablish normal bladder function and capacity.

On discharge, a client who underwent left modified radical mastectomy expresses relief that "the cancer" has been treated. When discussing this issue with the client, the nurse should stress that she:

should continue to perform breast self-examination on her right breast.

A client with urinary retention needs to undergo a procedure to insert an indwelling catheter. Before catheterization, the nurse would discuss with the physician information about

the type and size of the catheter to be used. Before catheterization, the nurse should inquire about the type and size of the catheter to be used and whether the catheter should be removed or retained in place after the bladder is empty. Inserting a nasogastric tube, administering enemas, and placing IV lines are measures taken during preoperative and postoperative preparation in the case of surgery.

A client is being discharged to home following a modified radical mastectomy. The nurse is providing discharge instructions and making arrangements for home care. Which of the following interventions will be included in her instructions? Select all that apply. a) Advise the client that blood pressure measurements, injections, blood donations, and IV infusions are lifelong restrictions on the side of the mastectomy. b) Encourage the client to wear restrictive clothing on the affected side. c) Recommend wearing gloves while doing yard or housework. d) Advise the client to use a disposable razor for shaving axillary hair.

• Advise the client that blood pressure measurements, injections, blood donations, and IV infusions are lifelong restrictions on the side of the mastectomy. • Recommend wearing gloves while doing yard or housework. Explanation: Because of impaired lymphatic flow, blood pressure measurements, injections, blood donations, and IV infusions in the affected arm are contraindicated. Gloves are worn to prevent injuries that may heal slowly or become infected. Advise the use of an electric razor for shaving axillary hair because it reduces the likelihood of cutting the skin, leading to infection. Discourage sleeping on the affected arm or wearing constrictive clothing that impairs circulation.

Nursing students are reviewing various procedures that can be used to obtain a tissue biopsy of the breast. They demonstrate understanding of the material when they identify which of the following as being done using local anesthesia and intravenous (IV) sedation? Select all that apply.

• Excisional biopsy • Wire needle localization

A client diagnosed with breast cancer has developed neutropenia secondary to chemotherapy. Which of the following would the nurse anticipate as being ordered to address the neutropenia? Select all that apply.

• Filgrastim • Pegfilgrastim

Which of the following are nursing interventions related to a patient with breast cancer? Select all that apply.

• Patient education and preparation for treatment • Promotion of positive body image • Relieving fear • Management of complications

Which of the following involves implantation of interstitial radioactive seeds under anesthesia to treat prostate cancer?

Brachytherapy

A client has had a right modified radical mastectomy and axillary lymph node dissection. The nurse is teaching the client about measures to reduce the risk of complications. The client demonstrates understanding of the instructions when she states which of the following? a) "I need to use an electric shaver when shaving my right armpit." b) "I can lift with my right arm objects that weigh as much as 15 pounds." c) "I should tell my manucurist that it is okay to trim the cuticles on my right hand." d) "Anytime I need blood drawn, they should get the sample from my right arm."

"I need to use an electric shaver when shaving my right armpit." To prevent complications after a right modified radical mastectomy and axillary lymph node dissection, the client should shave her right axillary area with an electric razor rather than a straight razor to reduce the risk of trauma to or breaks in the skin. Blood sampling should be done with the client's left arm, not the right one. The client should not lift objects more than 5 to 10 pounds with the right arm, and cuticles on the affected hand should be pushed back, not cut.

After teaching about self-care measures to a client who has been diagnosed with prostatitis, the nurse determines that the teaching was successful when the client states which of the following?

"I should avoid fluids like coffee and tea."

The nurse is demonstrating the technique for performing a testicular self examination (TSE) to a group of men for a company health fair. One of the men asks the nurse at what age a man should begin performing TSE. What is the best answer by the nurse?

"It should begin in adolescence."

A nurse is meeting with a woman scheduled to have a modified radical mastectomy to remove an aggressive breast tumor. The woman tells the nurse that she agreed to have the surgery before considering alternative options. Which of the following statements is the nurse's best response? a) "If I were you, I would consider a second opinion." b) "You might want to consider a less invasive surgical procedure." c) "You have a very competent surgeon and you should move forward as planned." d) "Tell me more about your fears and concerns."

"Tell me more about your fears and concerns." The type of surgery recommended depends on the stage of the tumor and the client's informed decision about treatment options. The client should be encouraged to express her concerns. Surgery should not be performed until the client is comfortable with the scheduled procedure. The type of surgery recommended depends on the stage of the tumor. A less invasive procedure may not remove all of the affected tissue. The nurse should not share her opinion with the client but rather support the client in making the best decision.

An 80-year-old client is being seen by the urologist because of erectile dysfunction. This client has a history of hypertension and heart disease. Which of the following would be the most likely cause of his erectile dysfunction?

All options are correct.

Which of the following patients is most likely to develop prostate cancer according to a nurse working at a health screening at the local mall?

A 56-year-old African American man

During a breast examination, which finding most strongly suggests that a client has breast cancer? a) Slight asymmetry of the breasts b) A fixed nodular mass with dimpling of the overlying skin c) Multiple firm, round, freely movable masses that change with the menstrual cycle d) Bloody discharge from the nipple

A fixed nodular mass with dimpling of the overlying skin Correct Explanation: A fixed nodular mass with dimpling of the overlying skin is common during late stages of breast cancer. Many women have slightly asymmetrical breasts. Bloody nipple discharge is a sign of intraductal papilloma, a benign condition. Multiple firm, round, freely movable masses that change with the menstrual cycle indicate fibrocystic breasts, a benign condition.

A 54-year-old woman presents to her healthcare provider's office where you practice nursing. She is very concerned that she might have breast cancer, especially after caring for her sister that recently died from the disease. Included in your discussion is the primary and most common sign of breast cancer. Which of the following would meet this criterion? a) A painless mass in the breast, most often in the upper outer quadrant b) A painful mass in the breast, most often in the lower quadrant near the nipple c) A painless mass in the breast, most often in the lower quadrant near the nipple d) A painful mass in the breast, most often in the upper outer quadrant

A painless mass in the breast, most often in the upper outer quadrant Correct Explanation: The primary sign of breast cancer is a painless mass in the breast, most often in the upper outer quadrant.

A patient has returned to the floor after undergoing a transurethral resection of the prostate (TURP). The patient has a continuous bladder irrigation system in place. The patient tells you he is experiencing bladder spasms and asks what you can do to relieve his discomfort. What is the most appropriate nursing action to relieve the discomfort of the patient?

Administer a smooth-muscle relaxant as ordered.

A 67-year-old client underwent a lumpectomy for a breast lesion that was determined to be malignant. Which of the following are factors in the client's history that may have increased her risk of breast cancer? a) Not giving birth b) Obesity c) Increased age d) All options are correct.

All options are correct. The risk for breast cancer in women increases with age. Certain factors appear to increase the risk of breast cancer. Being female, being older than 50 years of age, and having a family history of breast cancer are the most common risk factors. Additional factors include obesity, and having no children or having children after 30 years of age. The risk for breast cancer in women increases with age. Certain factors appear to increase the risk of breast cancer including obesity and having no children or having children after 30 years of age.

Kara Carpenter is a 54-year-old woman who just had a left radical mastectomy. The nurse caring for her is providing information on complications that may arise due to removing the axillary lymph nodes. Which of the following would not be included? a) Reduced range of motion b) Infection c) Tissue necrosis d) All would be included in the discussion.

All would be included in the discussion. Impaired lymphatic circulation predisposes to disfigurement, reduced range of motion, heaviness of the limb, skin changes, infection, and, in severe cases, tissue necrosis that may require amputation of the limb. Reduced range of motion is a potential consequence from removing the axillary lymph nodes. Tissue necrosis that may require amputation of the limb is a potential consequence from removing the axillary lymph nodes. Infection is a potential consequence from removing the axillary lymph nodes.

A client returns to the recovery room following a mastectomy. An initial postoperative assessment is performed by the nurse. What is the nurse's priority assessment? a) Checking level of pain first upon the clients return from the operating room. b) Assessing the vital signs and oxygen saturation levels. c) Checking for urinary retention and the need to void. d) Checking the dressing, drain, and amount of drainage.

Assessing the vital signs and oxygen saturation levels. The nurse prioritizes vital signs and breathing based on principles of ABCs.

A female client is diagnosed with breast abscess. She would like to continue to breast-feed her newborn. Which of the following would be most appropriate in this situation? a) Reduce the frequency of removing and reapplying the dressings. b) Assist the client to pump the breasts to remove breast milk. c) Instruct the client to wear a tight-fitting bra. d) Encourage the client to include protein content in the diet.

Assist the client to pump the breasts to remove breast milk. The nurse should help the client pump the breasts and remove breast milk to prevent engorgement. Because the client has decided to continue breastfeeding, the client should wear a loose-fitting bra. Including protein content in the diet would be unrelated to the client's current situation. Frequency of dressing changes does not play a role in the intervention

A physician explains to the patient that he has an inflammation of the Cowper glands. Where are the Cowper glands located?

Below the prostate, within the posterior aspect of the urethra

A client expresses concerns about future reproduction after a surgery to correct the cancer of the testes and for whom treatment has proceeded without collecting and storing sperm. Which of the following alternatives should a nurse suggest to the client? Choose all correct responses.

Become a foster parent Adoption Donor insemination

After teaching a group of students about the signs and symptoms of breast cancer, the instructor determines that additional teaching is needed when the group identifies which of the following? a) Painless mass b) Peau d'orange skin c) Nipple retraction d) Breast symmetry

Breast symmetry The primary sign of breast cancer is a painless mass in the breast. Other signs of breast cancer include a bloody discharge from the nipple, a dimpling of the skin over the lesion, retraction of the nipple, peau d'orange (orange peel) appearance of the skin, and a difference in size between the breasts.

After teaching a group of students about the signs and symptoms of breast cancer, the instructor determines that additional teaching is needed when the group identifies which of the following? a) Breast symmetry b) Peau d'orange skin c) Painless mass d) Nipple retraction

Breast symmetry Correct Explanation: The primary sign of breast cancer is a painless mass in the breast. Other signs of breast cancer include a bloody discharge from the nipple, a dimpling of the skin over the lesion, retraction of the nipple, peau d'orange (orange peel) appearance of the skin, and a difference in size between the breasts.

A nurse is providing care for a patient who has recently been admitted to the postsurgical unit from PACU following a transuretheral resection of the prostate. The nurse is aware of the nursing diagnosis of Risk for Imbalanced Fluid Volume. In order to assess for this risk, the nurse should prioritize what action?

Closely monitoring the input and output of the bladder irrigation system

A 57-year-old client is seen at the free clinic. He has an edematous glans penis, pain, and an extremely constricted foreskin. What treatment would you expect the physician to prescribe?

Circumcision

The nurse has identified the following nursing diagnosis on the plan of care for a client who has undergone breast cancer surgery: Disturbed sensory perception related to nerve irritation in the affected breast area. Which of the following indicates that the outcome has been achieved? a) Client identifies appropriate measures to reduce the risk of lymphedema. b) Client demonstrates appropriate use of prescribed analgesic. c) Client reports a sensation of pulling in the breast area. d) Client states that feeling in her breast area will gradually subside with time.

Client states that feeling in her breast area will gradually subside with time. Correct Explanation: After breast surgery, the nerves in the skin and axilla are often cut or injured, leading the client to experience various sensations, including tenderness, soreness, numbness, tightness, pulling, and twinges. These sensations are normal and usually persist for several months and then subside. Therefore, acknowledging that the feeling in her breast area will gradually subside indicates that she understands why the sensation is occurring and that it is normal. The report of the sensation of pulling indicates that the client is feeling the sensation, but it does not indicate whether the nursing diagnosis was addressed. Analgesics usually would not be required for these sensations. These sensations are not associated with lymphedema.

A 43-year-old client is a single parent and has been admitted for a left mastectomy after confirmation of cancer from a node biopsy. She has a daughter who is 12 years old. What are primary issues for the nurse to discuss with this client? a) Effect of surgery on the family's coping abilities. b) History of breast cancer in the family. c) How body image changes will affect her sexual relationships. d) Concerns regarding the cancer and how the surgery will affect her.

Concerns regarding the cancer and how the surgery will affect her. The two primary concerns are the confirmation of cancer and the impending mastectomy. The other issues are important, but not as high a priority at this time.

A patient has just returned to the floor following a transurethral resection of the prostate. A triple-lumen indwelling urinary catheter has been inserted for continuous bladder irrigation. What, in addition to balloon inflation, are the functions of the three lumens?

Continuous inflow and outflow of irrigation solution

The nurse working on a woman's cancer treatment floor performs nursing assessments on her assigned patients. It is most important for the nurse to report which of the following assessment findings? a) Immediate postoperative reports of throat tenderness b) Coolness and mottling of a newly constructed breast site c) Temperature of 99.2°F, pulse 72, respirations 18, blood pressure 130/80 d) Small amount of bloody drainage on surgical dressing 12 hours postoperatively

Coolness and mottling of a newly constructed breast site Correct Explanation: Mottling or an obvious decrease in skin temperature may signify flap loss and needs to be reported to the surgeon immediately. Throat discomfort immediately following surgery is an expected effect of airway management during surgery. A small amount of bloody drainage is an expected finding 12 hours postoperatively. Vital signs are within acceptable range for a postoperative patient.

Which of the following is a failure of one or both or both of the testes to descend into the scrotum?

Cryptorchidism

A 32-year-old client is concerned with the lumps that have developed in her breasts and is fearful of cancer. She reports variability in the size of the lumps. What could be causing her condition? a) Cyclical hormonal changes b) Progesterone c) Caffeine d) Nicotine

Cyclical hormonal changes Correct Explanation: Fibrocystic disease results from hormonal changes during the menstrual cycle. The likely cause is fibrocystic disease, which results from hormonal changes during the menstrual cycle.

A 29-year-old patient has just been told that he has testicular cancer and needs to have surgery. During a presurgical appointment, the patient admits to feeling devastated that he requires surgery, stating that it will leave him "emasculated" and "a shell of a man." The nurse should identify what nursing diagnosis when planning the patient's subsequent care?

Disturbed Body Image Related to Effects of Surgery

A patient experiences hypotension, lethargy, and muscle spasms while receiving bladder irrigations after a transurethral resection of the prostate (TURP). What is the first action the nurse should take

Discontinue the irrigations.

A patient is planning to use a negative-pressure (vacuum) device to maintain and sustain an erection. What should the nurse caution the patient about with the use of this device?

Do not leave the constricting band in place for longer than 1 hour to avoid penile injury.

The nurse is providing preoperative care to a client who is to undergo a radical prostatectomy for prostate cancer. The client's plan of care includes the nursing diagnosis of "anxiety related to surgery and its outcome." Which of the following would be most appropriate for the nurse to do?

Ensure privacy when discussing sensitive or embarrassing sexual issues.

A client with a penile implant is to be informed about the possible complications after discharge. Which of the following are complications that occur due to an incorrectly-sized implant, pressure, or friction of the implanted cylinders?

Erosion of penile or urethral tissue

Which of the following is the most common organism implicated in prostatitis?

Escherichia coli

A 48-year-old man recently diagnosed with benign prostatic hyperplasia (BPH), reports taking Serenoa repens (saw palmetto berry). The nurse needs to intervene if the physician orders which of the following treatments?

Finasteride (Proscar)

Which of the following is a characteristic of a breast cancer mass? a) Firm, hard, embedded in surrounding tissue b) Occurs as disseminated masses c) Symmetrical mass d) Tender upon palpation

Firm, hard, embedded in surrounding tissue Correct Explanation: A characteristic of a breast cancer mass is a firm, hard, embedded lesion in surrounding tissue. It has an irregular shape, usually not tender, and occurs as a single mass in one breast.

What should a male client older than age 50 do to help ensure early identification of prostate cancer?

Have a digital rectal examination and prostate-specific antigen (PSA) test done yearly.

Which of the following is a part of the client teaching that help the nurse assist a client following treatment for cancer of the prostate gland to manage and minimize the possibility of a recurrence of the primary cancer or metastasis?

Have regular PSA levels and repeat lymph node biopsies

Which of the following is a component of the patient teaching that helps the nurse assist a patient following treatment for cancer of the prostate gland to manage and minimize the possibility of a recurrence of the primary cancer or metastasis?

Have regular prostate-specific antigen (PSA) levels tested and repeat lymph node biopsies.

An adolescent is identified as having a collection of fluid in the tunica vaginalis of his testes. The nurse knows that this adolescent will receive what medical diagnosis?

Hydrocele

A 65-year-old man complains to his health care provider that, when he urinates, he has to start and stop several times over a period of minutes in order to fully empty his bladder. The nurse is aware that this is not uncommon in men over the age of 60. This "double voiding" is directly related to which of the following?

Hyperplasia of the prostate gland

A patient has presented at the clinic with symptoms of benign prostatic hyperplasia. What diagnostic findings would suggest that this patient has chronic urinary retention?

Increased blood urea nitrogen (BUN)

The nurse is caring for a client who is beginning doxorubicin (Adriamycin) therapy for breast cancer. When preparing the client for probable side effects, which would the nurse include? a) Information regarding wigs from the American Cancer Society b) Information regarding depression from a mental health association c) Information regarding high caloric meals from a dietician d) Information about blood donation from the American Red Cross

Information regarding wigs from the American Cancer Society Alopecia is a common side effect from the use of doxorubicin (Adriamycin). It is best for the client to be proactive in planning for hair loss so that the client has a suitable plan. Also, the American Cancer Society in some areas offers financial support and guidance in obtaining a wig or head covering. Blood donation is not completed during chemotherapy because anemia is common. Due to recent antiemetic medications, nausea is less common not as prolonged. Client's rarely need a high-calorie diet. Emphasis is placed on nutrient dense, not calorie dense. Depression may be a concern, and a support group is an excellent resource. A mental health association may be more than what is needed.

A patient is suspected to have prostate cancer related to observed clinical symptoms. What definitive test can the nurse assist with to confirm a diagnosis of prostate cancer?

Prostate biopsy

When the female patient demonstrates a wartlike growth near the nipple, causing bloody nipple discharge, the patient is exhibiting signs of which disease process? a) Intraductal papilloma b) Fibroadenoma c) Paget's disease d) Acute mastitis

Intraductal papilloma Correct Explanation: Intraductal papilloma is a wartlike growth that often involves the large milk ducts near the nipple, causing bloody nipple discharge. Surgery usually involves removal of the papilloma and a segment of the duct where the papilloma is found. Paget's disease is a malignancy of mammary ducts with early signs of erythema of nipple and areola. Acute mastitis is demonstrated by nipple cracks or abrasions along with reddened and warm breast skin and tenderness. Fibroadenoma is characterized as the occurrence of a single, nontender mass that is firm, mobile, and not fixed to breast tissue or chest wall.

Which of the following is accurate regarding sildenafil (Viagra)?

Its side effects include headache, flushing, and dizziness.

The nurse is caring for a patient who had a 1-day postoperative transurethral resection of the prostate (TURP). Which of the following assessment findings does the nurse expect? Select all that apply.

Large amounts of amber-colored urine in the drainage bag Reports of the urge to void from the patient Drainage tube secured to the inner thigh

A patient is 24 hours postoperative following prostatectomy and the urologist has ordered continuous bladder irrigation. What color of output should the nurse expect to find in the drainage bag?

Light pink

The nurse is providing care to a client who has had a transurethral resection of the prostate. The client has a three-way catheter drainage system in place for continuous bladder irrigation. The nurse anticipates that the catheter may be removed when the urine appears as which of the following

Light yellow and clear

The nurse is providing care to a client who has had surgery as treatment for breast cancer. The nurse would be alert for the development of which of the following? a) Breast abscess b) Fibroadenoma c) Lymphedema d) Fibrocystic breast disease

Lymphedema Correct Explanation: Lymphedema occurs in some women after breast cancer surgery. It causes disfigurement and increases the lifetime potential for infection and poor healing. Fibrocystic breast disease and fibroadenoma are two benign breast conditions that occur usually in premenopausal woman. Breast abscess is the infectious and inflammatory breast condition that is common among breast-feeding mothers.

Sentinel lymph node mapping is done to validate the lack of lymph node metastasis. Which of the following complications does this technique help avoid? a) Lymphedema b) Fibroadenoma c) Breast cancer d) Mastalgia

Lymphedema Correct Explanation: Validating the lack of lymph node metastasis allows the surgeon to preserve more breast and axillary tissue and chest muscle. Leaving more normal lymph nodes intact reduces the potential for complications, such as lymphedema caused by the extensive disruption of lymphatic circulation.

A client is considering breast augmentation. Which of the following would the nurse recommend to the client to ensure that there are no malignancies? a) Breast biopsy b) Mammogram c) Mastopexy d) Ultrasound

Mammogram Correct Explanation: When caring for a client considering breast augmentation, the nurse should provide her with a general guideline to have a mammogram to verify that there are no malignancies. Mastopexy involves a breast life for drooping breasts. Ultrasound or breast biopsy would not be necessary unless there was evidence of a problem.

Which of the following terms refers to breast pain? a) Mastalgia b) Mammoplasty c) Mastitis d) Gynecomastia

Mastalgia Mastalgia refers to breast pain. Mastitis is an inflammation or infection of the breast. Gynecomastia refers to overdeveloped breast tissue typically seen in adolescent boys. Mammoplasty refers to a surgical procedure to reconstruct or change the size or shape of the breast.

Which of the following would be inconsistent as a risk factor for breast cancer? a) Family history of breast cancer b) Increased age c) Late menopause d) Multiparity

Multiparity Correct Explanation: Nulliparity is a risk factor for breast cancer, along with late menopause, increased age, and family history of breast cancer.

The nurse is caring for a group of breast cancer survivors post mastectomy. When developing a list of instructions of points to avoid, which point is highlighted? a) Applying cream to breast b) Arm exercises on affected side c) Wearing loose fitting shirts d) No lifting greater than 15 lb

No lifting greater than 15 lb Correct Explanation: Of the list citing things to avoid, the correct option to avoid is lifting more than 15 lb. This fact is important and needs to be highlighted. Tight-fitting or constrictive clothing is to be avoided. The client is encouraged to place cream on any area of the breast which is dry. Arm exercises are routinely completed on the affected arm.

The nurse teaches the female patient who is premenopausal to perform breast self-examination (BSE) at which time frame? a) With the onset of menstruation b) On day 5 to day 7, counting the first day of menses as day 1 c) On day 2 to day 4, counting the first day of menses as day 1 d) Any time during the month

On day 5 to day 7, counting the first day of menses as day 1 BSE is best performed after menses, when less fluid is retained. Because most women notice increased tenderness, lumpiness, and fluid retention before their menstrual period, BSE is not recommended with the onset of menses. Because the tenderness, lumpiness, and fluid retention problems noticed by women in relation to onset of menses and generally continues through menses, BSE is not recommended during that time. Because most women notice increased tenderness, lumpiness, and fluid retention before their menstrual period, BSE is best performed when the time for menses is taken into account.

A 59-year-old African American male has opted for the hormonal androgen deprivation therapy (ADT) to treat his prostate cancer. Which of the following surgical procedures is the patient most likely to undergo?

Orchiectomy

The nurse is completing the admission history for a client who is admitted for a reduction mammoplasty. Which of the following client statements is uncommon when explaining the rationale for the procedure? a) Back pain b) Skin irritation c) Low self-esteem d) Others disapprove

Others disapprove Correct Explanation: The rationale for a reduction mammoplasty most often comes from the client herself as she is experiencing a complication due to the size of the breast. Back pain, low self-esteem and a self-consciousness, and skin irritation are common rationales.

A nurse is assessing a patient who presented to the ED with priapism. The student nurse is aware that this condition is classified as a urologic emergency because of the potential for what?

Permanent vascular damage

An uncircumcised 78-year-old male has presented at the clinic complaining that he cannot retract his foreskin over his glans. On examination, it is noted that the foreskin is very constricted. The nurse should recognize the presence of what health problem?

Phimosis

A nurse practitioner is assessing a 55-year-old male patient who is complaining of perineal discomfort, burning, urgency, and frequency with urination. The patient states that he has pain with ejaculation. The nurse knows that the patient is exhibiting symptoms of what?

Prostatitis

A 22-year-old male is being discharged home after surgery for testicular cancer. The patient is scheduled to begin chemotherapy in 2 weeks. The patient tells the nurse that he doesn't think he can take weeks or months of chemotherapy, stating that he has researched the adverse effects online. What is the most appropriate nursing action for this patient at this time?

Provide empathy and encouragement in an effort to foster a positive outlook.

Ernst Weber, a 73-year-old retired carpenter, has been diagnosed with epididymitis after examination and diagnostics ordered by a physician with the urology group where you practice nursing. What would you expect to find in the lab results from his urinalysis?

Pus and bacteria

On a follow-up visit, the patient is informed that her breast cancer has recurred. The nurse knows which of the following situations places the patient at risk for developing bone metastases? a) Recurrence within 2 years of the original diagnosis b) No maternal family history of breast cancer c) Aggressive treatment, including surgery, radiation, and hormonal therapy d) Previous therapeutic response to chemotherapy

Recurrence within 2 years of the original diagnosis Local recurrence may be an indicator that systemic disease will develop in the future, particularly if it occurs within 2 years of the original diagnosis. Local recurrence in the absence of systemic disease is treated aggressively with surgery, radiation, and hormonal therapy. Overall prognosis and optimal treatment are determined by a variety of factors such as the time to recurrence from the original diagnosis and history of prior treatments.

A nurse is caring for a client who has just had a modified radical mastectomy with immediate reconstruction. She's in her 30s and has two young children. Although she's worried about her future, she seems to be adjusting well to her diagnosis. What should the nurse do to support her coping? a) Encourage the client to proceed with the next phase of treatment. b) Tell the client's spouse or partner to be supportive while she recovers. c) Recommend that the client remain cheerful for the sake of her children. d) Refer the client to the American Cancer Society's Reach for Recovery program or another support program.

Refer the client to the American Cancer Society's Reach for Recovery program or another support program. The client isn't withdrawn and doesn't show other signs of anxiety or depression. Therefore, the nurse can probably safely approach her about talking with others who have had similar experiences, either through Reach for Recovery or another formal support group. The nurse may educate the client's spouse or partner and listen to his concerns, but the nurse shouldn't tell the client's spouse what to do. The client must consult with her physician and make her own decisions about further treatment. The client needs to express her sadness, frustration, and fear. She can't be expected to be cheerful at all times.

A client underwent a transurethral resection of the prostate gland 24 hours ago and is on continuous bladder irrigation. Which nursing intervention is appropriate?

Use sterile technique when irrigating the catheter.

Which of the following herbal remedies is used to treat symptoms of benign prostatic hypertrophy (BPH)?

Saw palmetto

A patient is having a DRE in the physician's office and the nurse is to assist in the examination. What can the nurse instruct the client to do to decrease the discomfort from the exam?

Take a deep breath and exhale when the physician inserts a gloved finger into the rectum.

Which of the following should be included as part of the home care instructions for a patient with epididymitis and orchitis?

Take prescribed antibiotics

A patient comes to the emergency department and tells the nurse, "I took a pill to help me perform and then passed out." The nurse is assessing the patient and finds a nitroglycerin patch on his back. What is the first intervention the nurse must perform?

Take the patient's blood pressure.

Students are reviewing information about the use of adjuvant hormonal therapy for breast cancer. They demonstrate understanding of this information when they identify which of the following as an example of a selective estrogen receptor modulator (SERM)? a) Tamoxifen b) Exemestane c) Letrozole d) Anastrozole

Tamoxifen Tamoxifen is an example of a SERM. It acts by competing for estrogen-receptor binding sites. Anastrozole, letrozole, and exemestane are examples of aromatase inhibitors, drugs that block estrogen production.

A public health nurse is teaching a health class for the male students at the local high school. The nurse is teaching the boys to perform monthly testicular self-examinations. What point would be appropriate to emphasize?

Testicular cancer is a highly curable type of cancer.

A client comes to the clinic for a routine evaluation. During the physical examination, the nurse palpates the client's breast and finds a small lump. Which of the following would lead the nurse to suspect possible breast cancer? a) The lump is soft. b) The client reports tenderness during the palpation. c) The lump is irregularly shaped. d) The lump is mobile.

The lump is irregularly shaped. Correct Explanation: Generally, breast cancer lesions are nontender, fixed rather than mobile, and hard with irregular borders. Diffuse breast pain and tenderness with menstruation are usually associated with benign breast disease.

The client arrives at a public health clinic worried that she has breast cancer since finding a lump in her breast. When assessing the breast, which assessment finding is characteristic of fibrocystic disease? a) One breast is larger than the other. b) Nipple retractions are noted. c) The lump is round and movable. d) The lump is firm and nonmovable.

The lump is round and movable. When assessing a breast with fibrocystic disease, the lumps typically are different from cancerous lumps. The characteristic breast mass of fibrocystic disease is soft to firm, circular, movable, and unlikely to cause nipple retraction. A cancerous mass is typically irregular in shape, firm, and nonmovable. Lumps typically do not make one breast larger than the other. Nipple retractions are suggestive of cancerous masses.

A man comes to the clinic complaining that he is having difficulty obtaining an erection. When reviewing the patient's history, what might the nurse note that contributes to erectile dysfunction?

The patient has a history of hypertension.

A nurse is performing an admission assessment on a 40-year-old man who has been admitted for outpatient surgery on his right knee. While taking the patient's family history, he states, "My father died of prostate cancer at age 48." The nurse should instruct him on which of the following health promotion activities?

The patient should have PSA levels drawn regularly.

The nurse is caring for a client who is ordered a sentinel lymph node biopsy. The physician explained the procedure and desired outcome. Which statement, made by the client, indicates a need for further instruction? a) The procedure allows for conservation of breast tissue. b) The procedure allows for an understanding of the spread of cancer cells. c) The procedure removes all cancer from the body. d) The procedure includes minimal surrounding tissue damage.

The procedure removes all cancer from the body. Explanation: Sentinel lymph node mapping involves identifying the first (sentinel) lymph nodes through which the breast cancer cells would spread to regional lymph nodes in the axilla. Validating the lack of lymph node metastasis allows the surgeon to preserve more breast tissue, axillary tissue and chest muscle. Further instruction would be needed to explain that the sentinel lymph node biopsy does not remove cancer from the body.

A client complains of having tender and painful breasts, often feeling multiple lumps within her breast tissue. The nurse would need to gather additional information about which of the following? a) Client's workplace in relation to the surroundings b) Timing of symptoms in relation to the menstrual cycle c) Bathing frequency and living surroundings d) Alcohol and caffeine consumption

Timing of symptoms in relation to the menstrual cycle Considering that the client has tender and painful breasts and that she often feels lumps within her breast tissue, it is most likely that she suffers from fibrocystic breast disease. To confirm these findings, the nurse should ask relevant questions about the characteristics and timing of symptoms in relation to the menstrual cycle. Symptoms of fibrocystic breast disease are noticeable before menstruation and usually abate during menstruation. The size of the cyst becomes larger before menstruation and often changes with the menstrual cycle. The nurse should further ask the client about her habits of smoking and consuming coffee, chocolate, and caffeinated soft drinks, not alcohol, because they aggravate the condition. Workplace surroundings or cleanliness habits do not matter because fibrocystic breast disease is not infectious.

A client complains of having tender and painful breasts, often feeling multiple lumps within her breast tissue. The nurse would need to gather additional information about which of the following? a) Bathing frequency and living surroundings b) Client's workplace in relation to the surroundings c) Alcohol and caffeine consumption d) Timing of symptoms in relation to the menstrual cycle

Timing of symptoms in relation to the menstrual cycle Correct Explanation: Considering that the client has tender and painful breasts and that she often feels lumps within her breast tissue, it is most likely that she suffers from fibrocystic breast disease. To confirm these findings, the nurse should ask relevant questions about the characteristics and timing of symptoms in relation to the menstrual cycle. Symptoms of fibrocystic breast disease are noticeable before menstruation and usually abate during menstruation. The size of the cyst becomes larger before menstruation and often changes with the menstrual cycle. The nurse should further ask the client about her habits of smoking and consuming coffee, chocolate, and caffeinated soft drinks, not alcohol, because they aggravate the condition. Workplace surroundings or cleanliness habits do not matter because fibrocystic breast disease is not infectious.

A client with a fibroadenoma is being scheduled for diagnostic testing. Which of the following would the nurse expect as most likely? a) Ultrasound b) Culture of discharge c) Excisional biopsy d) Mammogram

Ultrasound Explanation: Ultrasound can reveal physical characteristics unique to a fibroadenoma versus malignant mass with a higher degree of accuracy than mammography. In the case of very young women—an atypical age for breast cancer—an excisional biopsy is performed only if the mass changes or becomes larger. If the mass is detected in a woman with a higher risk for developing breast cancer, such as one with a family history or of an older age, a biopsy is performed to confirm that the tissue is indeed benign. There is no discharge to culture.

A group of students are reviewing the anatomy and physiology of the breasts. The students demonstrate understanding of breast structure when they identify the tail of Spence as an extension of which quadrant? a) Lower inner b) Upper inner c) Lower outer d) Upper outer

Upper outer The tail of Spence is an area of breast tissue that extends from the upper outer quadrant of the breast into the axilla. The upper and lower inner quadrants are closer to the midline. The lower outer quadrant is below the upper outer quadrant.

Which of the following instructions regarding future sexual activity should a nurse give a client with a vasectomy?

Use a reliable method of contraception until the physician ensures that sperm no longer are present.

A nurse is instructing a premenopausal woman about breast self-examination. The nurse should tell the client to do her self-examination: a) on the same day each month. b) immediately after her menses. c) on the first day of the menstrual cycle. d) at the end of her menstrual cycle.

immediately after her menses. Premenopausal women should do their self-examination immediately after the menstrual period, when the breasts are least tender and least lumpy. On the first and last days of the cycle, the woman's breasts are still very tender. Postmenopausal women, because their bodies lack fluctuation of hormone levels, should select one particular day of the month to do breast self-examination.

On discharge, a client who underwent left modified radical mastectomy expresses relief that "the cancer" has been treated. When discussing this issue with the client, the nurse should stress that she: a) should schedule a follow-up appointment in 6 months. b) is lucky that the cancer was caught in time. c) should continue to perform breast self-examination on her right breast. d) will have irregular menses.

should continue to perform breast self-examination on her right breast. Having breast cancer on her left side puts the client more at risk for cancer on the opposite side and chest wall. Therefore, the nurse should stress the importance of monthly breast self-examinations and annual mammograms. Although the tumor was found, it was large enough to require a mastectomy, and could put the patient at risk for metastasis. Follow-up appointments should be monthly for the first few months and then scheduled at the direction of her health care provider. Modified radical mastectomy shouldn't affect the menstrual cycle.

Testicular cancer risk is highest for adolescents and men younger than age 35. To specifically address testicular cancer risk, a nurse should modify client teaching for male clients to include:

testicular self-examination.


Ensembles d'études connexes

Cisco Networking ITN Module 16 Review

View Set

History of the Holocaust Week 4 Terms

View Set

CIS121 Programming and Logic Chapter 4

View Set